SlideShare a Scribd company logo
1 of 49
Download to read offline
S&S-8 (The z-transform) 1 of 49 Dr. Ravi Billa
Signals & Systems – 8 August 5, 2011
VIII. The z-transform
Syllabus: Fundamental difference between continuous and discrete time signals, Discrete time signal
representation using complex exponential and sinusoidal components, Periodicity of discrete time
signal using complex exponential signal, Concept of z-transform of a discrete sequence, Distinction
between Laplace, Fourier and z transforms, Region of convergence in z-transform, Constraints on ROC
for various classes of signals, Inverse z-transform, Properties of z-transforms.
O&W Ch. 10.
Contents:
8.1 Continuous-time signals
8.2 Discrete-time signals
8.3 The z-transform
8.4 Transforms of some useful sequences
8.5 Important properties of z-transforms
8.6 Transforms of some useful sequences, cont’d.
8.7 Region of convergence (ROC)
8.8 Inverse z-transform by partial fractions
8.9 Inverse z-transform by power series expansion (long division)
www.jntuworld.com
www.jntuworld.com
S&S-8 (The z-transform) 2 of 49 Dr. Ravi Billa
8.1 Continuous-time signals
Continuous-time signals are functions of the continuous time variable, t. In this course the
independent variable is time; however, this need not always be the case.
Example 1 [Sinusoidal signal] The household voltage in India is 230 Volts (rms), 50 Hertz or cycles
per second. The waveform may be expressed in various forms, for example,
( ) √ , or ( ) √
The period is 1/50 second. The amplitude is √ volts, the radian frequency is rad/sec., the
Hertz (or cyclic) frequency is = 50 Hz.
Quiz What are the amplitude, rms value, period and frequency of ( ) ?
Periodic and aperiodic signals A signal ( )is periodic if and only if
( ) ( )
where is the period. The smallest value such that the above equation is satisfied is called the
fundamental period or simply the period. Any deterministic signal not satisfying the above equation is
called aperiodic. If a signal is periodic with a fundamental period of , then it is also periodic with a
period of , … or any integer multiple of .
Note We use the symbols (Hz) and (radians/second) for analog frequencies.
Example 2 [Periodicity] For the signal
( ) ( ) ( )
the parameter A is the amplitude (peak value), F0 is the frequency (Hertz or cycles per second), and θ or
( ) is the phase. The radian frequency is rad/sec. The period is given by
⁄ .The periodicity of the above ( )– whether it is periodic or not – can be verified by
checking if
( ( )) ( )
Now, replacing by , we have
( ( )) ( ( ) ) ( )
[ ( ) ( ) ]
( ) ( )
Note that is the smallest value of such that ( ) ( ). There are no restrictions
on the period ( ) or the frequency ( ) in this analog case (as there are in the case of the discrete-
time counterpart).
The sum of two or more continuous-time sinusoids may or may not be periodic, depending on the
relationships among their respective periods (or frequencies). If the ratio of their periods (or
frequencies) can be expressed as a rational number, their sum will be a periodic signal. For instance, the
sum of two sinusoids of frequencies and is periodic if ⁄ = ⁄ where the ’s are integers.
www.jntuworld.com
www.jntuworld.com
S&S-8 (The z-transform) 3 of 49 Dr. Ravi Billa
Another way of putting this is: if their frequencies are commensurable, their sum will be a periodic
signal. Two frequencies, F1 and F2, are commensurable if they have a common measure. That is, there
is a number contained in each an integral number of times. Thus, if is the smallest such number,
and
is called the fundamental frequency. The periods, and corresponding to and are
therefore related by
which is rational.
HW Determine if the signal ( ) ( ) ( ) is periodic and if so determine its
frequency. [See BasicSim-51]
Note that if the signals ( ) and ( ) have the same period , then ( ) ( ) ( ) also is
periodic with the same period . That is, linear operations (addition in this case) do not affect the
periodicity of the resulting signal. In contrast nonlinear operations, generally, can produce different
frequencies (or periodicities). See example below.
Example 3 [Product of two sinusoids] Multiplication is a nonlinear operation. If ( ) and
( ) , then the signal ( ) ( ) ( ) will contain the sum and difference frequencies,
( ) and ( ), and the original frequencies and are absent. A similar observation
holds for the signal ( ) = ( ) or ( ).
However, if ( ) and ( ) have different (commensurable) frequencies then a linear
combination may produce new frequency(s): an example is ( ) ( ) ( ).
HW Recast the linear sum signal ( ) ( ) ( ) as the product of two sinusoids.
[See BasicSim-51]
Real exponential signal Consider ( ) where and are constants. For example, ( )
or ( ) . Sketch the waveforms. How do you find the time constant? Put the exponential
in the form of . For example, ( ) .
Complex exponential signal In what follows we make extensive use of Euler’s theorem
and the corresponding formulae
( )⁄ ( )⁄
It is often mathematically convenient to represent real signals in terms of complex quantities. Given the
phasor (or complex number) | | | | , the real sinusoidal signal ( ) can be expressed in
terms of the complex signal ̃( ) . In particular ( ) is defined as the real part of ̃( )as
( ) {̃( )} { } ( )
{| | }
www.jntuworld.com
www.jntuworld.com
S&S-8 (The z-transform) 4 of 49 Dr. Ravi Billa
{| | ( )
}
| | ( )
The complex signal ̃( ) | | ( )
is referred to as a rotating phasor characterized by the three
parameters: (1) The amplitude, | |, (2) The phase, , and (3) The radian frequency . The
signal ̃( ) is periodic with period .
Alternatively, we may relate ̃( ) to its sinusoidal counterpart by writing
( )
̃( ) ̃ ( ) | | ( )
| | ( )
( )
which is a representation in terms of conjugate, oppositely rotating phasors.
The expressions (1) and (2) for ( ) | | ( ) are referred to as time-domain
representations.
An alternative representation for x(t) is provided in the frequency domain. Since ̃( )
| | ( )
is completely specified by | | and for a given value of (or ), this
alternative frequency domain representation can take the form of two plots, one showing the amplitude
| | as a function of frequency, , and the other showing the phase as a function of . The result is the
single-sided spectrum shown here.
Another version of the frequency domain representation, the two-sided spectrum, results if we
make a spectral plot corresponding to Eq. (2). The two-sided spectrum has lines at both and as
shown below. Note the even symmetry of the amplitude plot and the odd symmetry of the phase plot.
F
F0
Phase

–
–F0
|A|/2
F
–F0
Amplitude
|A|/2
F0

F0
Phase
F0
|A|
F
0
Amplitude
F0
www.jntuworld.com
www.jntuworld.com
S&S-8 (The z-transform) 5 of 49 Dr. Ravi Billa
8.2 Discrete-time signals
Discrete-time signals are functions of the discrete time variable, n. In this course the independent
variable is time; however, this need not always be the case.
If a continuous-time signal x(t) is sampled at T-second intervals, the result is a discrete-time
signal or sequence { ( )} where n is an index on the sampling instants. For convenience we shall
drop the T and the braces and use just x(n) to represent the sequence.
Example 1 Consider the continuous-time waveform with a frequency of 4 Hz given by
( )
Sampling the above signal at T-second intervals results in the discrete-time signal ( ).
Mathematically, this is accomplished by replacing by and the expression for ( ) is given by
( )
If, for instance, the sampling frequency is 16 Hz then T = 1/16 and the discrete-time signal becomes
( ) ( ⁄ ) ( )
In the notation ( ) we may drop the symbol T and refer to the discrete-time signal simply as
( ) ( ).
Quiz Write expressions for ( ) and ( ) obtained by sampling, respectively, ( )
and ( ) at 16 Hz. Sketch and label the sequences ( ), ( ) and ( ) for n = 0 to
12.
www.jntuworld.com
www.jntuworld.com
S&S-8 (The z-transform) 6 of 49 Dr. Ravi Billa
MATLAB plotting of continuous-time function
%Plot x1(t) = cos 2π4t as t goes from 0 to 0.5 sec (2 cycles) in steps of T = 1/160 sec.
t = 0: 1/160: 0.5; x1 = cos (2*pi*4*t); plot(t,x1);
xlabel ('Time, t, seconds'), ylabel('x1(t)');
title ('x1(t) = cos (2*pi*4*t) – 4Hz continuous Cosine plotted at T = 1/160 sec.')
HW Explain the effect of changing “plot(t, x1)” in the above program segment to “plot(t, x1, 'bo')”.
%Plot x1(t) = cos 2π4t as t goes from 0 to 0.5 sec (2 cycles) in steps of T = 1/160 sec.
t = 0: 1/160: 0.5; x1 = cos (2*pi*4*t); plot(t,x1, 'bo');
xlabel ('Time, t, seconds'), ylabel('x1(t)');
title ('x1(t) = cos (2*pi*4*t) – 4Hz continuous Cosine plotted at T = 1/160 sec.')
0 0.05 0.1 0.15 0.2 0.25 0.3 0.35 0.4 0.45 0.5
-1
-0.8
-0.6
-0.4
-0.2
0
0.2
0.4
0.6
0.8
1
Time, t, seconds
x1(t)
x1(t) = cos (2*pi*4*t) – 4Hz continuous Cosine plotted at T = 1/160 sec.
www.jntuworld.com
www.jntuworld.com
S&S-8 (The z-transform) 7 of 49 Dr. Ravi Billa
MATLAB plotting of discrete-time function
%Plot x1(n) = cos nπ/20 as n goes from 0 to 80 (2 cycles) in steps of 1 (T = 1/160 sec.)
n = 0: 1: 80; x1 = cos (n*pi/20); plot(n, x1, 'bo');grid %'bo' = Blue circles
xlabel ('Sample number, n'), ylabel('x1(n)');
title ('x1(n) = cos (n*pi/20) – 4Hz sampled Cosine at 160 samples/sec.')
HW Explain the result of changing “plot(n, x1, 'bo')” in the above program segment to “plot(n, x1)”.
%Plot x1(n) = cos nπ/20 as n goes from 0 to 80 (2 cycles) in steps of 1 (T = 1/160 sec.)
n = 0: 1: 80; x1 = cos (n*pi/20); plot(n, x1);grid
xlabel ('Sample number, n'), ylabel('x1(n)');
title ('x1(n) = cos (n*pi/20) – 4Hz sampled Cosine at 160 samples/sec.')
0 10 20 30 40 50 60 70 80
-1
-0.8
-0.6
-0.4
-0.2
0
0.2
0.4
0.6
0.8
1
Sample number, n
x1(n)
x1(n) = cos (n*pi/20) – 4Hz sampled Cosine at 160 samples/sec.
www.jntuworld.com
www.jntuworld.com
S&S-8 (The z-transform) 8 of 49 Dr. Ravi Billa
MATLAB plotting of discrete-time function – the stem plot At each sampling instant, n, a vertical
line (stem) is erected with a height equal to the sample value.
%Stem plot x1(n) = cos nπ/20 as n goes from 0 to 80 (2 cycles) in steps of 1
%(T = 1/160 sec.)
n = 0: 1: 80; x1 = cos (n*pi/20); stem(n,x1)
xlabel ('Sample number, n'), ylabel('x1(n)');
title ('x1(n) = cos (n*pi/20) – Stem plot of 4Hz Cosine sampled at 160 Hz.')
0 10 20 30 40 50 60 70 80
-1
-0.8
-0.6
-0.4
-0.2
0
0.2
0.4
0.6
0.8
1
Sample number, n
x1(n)
x1(n) = cos (n*pi/20) – Stem plot of 4Hz Cosine sampled at 160 Hz.
www.jntuworld.com
www.jntuworld.com
S&S-8 (The z-transform) 9 of 49 Dr. Ravi Billa
%Difference between plotting with 'bo' versus 'b'
n = 0: 1: 80;
%
%'bo' = Data points are marked with blue circles
x1 = cos (n*pi/20);
subplot(2, 1, 1), plot(n, x1, 'bo'); %subplot(2, 1, 1) – 2 rows, 1 column, #1
xlabel ('n'), ylabel('x1(n)'); title ('Sub-plot(2,1,1): 1st
Row, 1st
Column, Plot #1');
legend ('Legend 1'); grid;
%
%'b' = Data points are shown as a blue connected (continuous) line!!
subplot(2, 1, 2), plot(n, cos (n*pi/20), 'b'); %subplot(2, 1, 2) – 2 rows, 1 column, #2
xlabel ('n'), ylabel('x2(n)'); title ('Sub-plot(2,1,2): 2nd
Row, 1st
Column, Plot #2');
legend ('Legend 2'); %No grid
0 10 20 30 40 50 60 70 80
-1
-0.5
0
0.5
1
n
x1(n)
Sub-plot(2,1,1): 1st Row, 1st Column, Plot #1
Legend 1
0 10 20 30 40 50 60 70 80
-1
-0.5
0
0.5
1
n
x2(n)
Sub-plot(2,1,2): 2nd Row, 1st Column, Plot #2
Legend 2
www.jntuworld.com
www.jntuworld.com
S&S-8 (The z-transform) 10 of 49 Dr. Ravi Billa
Example 3 [The real exponential sequence] The continuous-time real-valued exponential function,
when sampled, results in the real-valued exponential sequence also known as a geometric series.
Consider the familiar continuous time signal defined for positive time
( )
The sampled version is given by setting t = nT
( ) ( )
Dropping the T from ( ) and setting we can write the positive time sequence as
( )
Using the unit step sequence ( ), the sequence ( ) may also be expressed as
( ) ( )
(The sequence can also be defined for both positive and negative n, by simply writing ( ) for all
n.)
0 1 2 3 4 5 6
x(n) = an
u(n)
n
1
a2
a3
a4
a a = T
e 
0 1 2 3 4 5 6
x(t) = t
e 
, t  0
t
1
www.jntuworld.com
www.jntuworld.com
S&S-8 (The z-transform) 11 of 49 Dr. Ravi Billa
MATLAB plots of ( ) = ( ) and ( )= ( )
K=2; alpha = 5; T=0.05;
%Continuous-time. Plot x(t) = K*exp(-alpha*t) for t = 0 to 1 sec.
t = 0: 1/100: 1;xt = K*exp(-alpha*t) ;
subplot(2,1,1);plot(t, xt, 'k');xlabel ('t'), ylabel('xt');
legend ('xt = K*exp(-alpha*t) for t = 0 to 1 sec.');
title ('Continuous-time'); grid;
%
%Discrete-time.Plotx(n) = K*exp(-alpha*n*T) for n = 0 to 20.
n = 0: 20; xn = K*exp(-alpha*n*T) ;
subplot(2,1,2);stem(n, xn, 'b'); xlabel ('n'), ylabel('xn');
legend ('xn = K*exp(-alpha*n*T) for n = 0 to 20');
title ('Discrete-time'); grid;
0 0.1 0.2 0.3 0.4 0.5 0.6 0.7 0.8 0.9 1
0
0.5
1
1.5
2
t
xt
Continuous-time
xt = K*exp(-alpha*t) for t = 0 to 1 sec.
0 2 4 6 8 10 12 14 16 18 20
0
0.5
1
1.5
2
n
xn
Discrete-time
xn = K*exp(-alpha*n*T) for n = 0 to 20
www.jntuworld.com
www.jntuworld.com
S&S-8 (The z-transform) 12 of 49 Dr. Ravi Billa
Example 4 [The exponential sequence, cont’d.] The real-valued exponential sequence x(n) = , is a
basic building block in discrete-time systems. If is a positive fraction then for n 0, the sequence
( ) is a monotonically decreasing sequence. If is a negative fraction then for n 0, the sequence
( ) is an alternating sequencewith decreasing magnitudes. See MATLAB segments below.
%Plot the function xn = a^n
a = 0.8; n = 0: 20;xn = a.^n;
subplot(2,1,1); stem(n, xn, 'b');xlabel ('n'), ylabel('x(n)');
title ('x(n) = a^n for n = 0 to 20');legend ('x(n) = (0.8)^n u(n)'); grid;
%
a = -0.8; n = 0: 20; xn = a.^n;
subplot(2,1,2); stem(n, xn, 'b'); xlabel ('n'), ylabel('x(n)');
title ('x(n) = a^n for n = 0 to 20'); legend ('x(n) = (-0.8)^n u(n)'); grid;
0 2 4 6 8 10 12 14 16 18 20
0
0.5
1
n
x(n)
x(n) = an
for n = 0 to 20
x(n) = (0.8)n
u(n)
0 2 4 6 8 10 12 14 16 18 20
-1
-0.5
0
0.5
1
n
x(n)
x(n) = an
for n = 0 to 20
x(n) = (-0.8)n
u(n)
www.jntuworld.com
www.jntuworld.com
S&S-8 (The z-transform) 13 of 49 Dr. Ravi Billa
Example 5 [The exponential sequence, cont’d.] If | |> 1 then for n 0, the sequence ( ) is a
monotonically increasing sequence if is positive. If, however, is negative then for n 0, the
sequence ( ) is an alternating sequencewith increasing magnitudes. See MATLAB segments below.
%Plot the function xn = a^n
a = 1.1; n = 0: 20; xn = a.^n;
subplot(2,1,1); stem(n, xn, 'b'); xlabel ('n'), ylabel('x(n)');
title ('x(n) = a^n for n = 0 to 20'); legend ('x(n) = (1.1)^n u(n)'); grid;
%
a = -1.1; n = 0: 20; xn = a.^n;
subplot(2,1,2); stem(n, xn, 'b'); xlabel ('n'), ylabel('x(n)');
title ('x(n) = a^n for n = 0 to 20'); legend ('x(n) = (-1.1)^n u(n)'); grid;
0 2 4 6 8 10 12 14 16 18 20
0
2
4
6
8
n
x(n)
x(n) = an
for n = 0 to 20
0 2 4 6 8 10 12 14 16 18 20
-10
-5
0
5
10
n
x(n)
x(n) = an
for n = 0 to 20
x(n) = (1.1)n
u(n)
x(n) = (-1.1)n
u(n)
www.jntuworld.com
www.jntuworld.com
S&S-8 (The z-transform) 14 of 49 Dr. Ravi Billa
Example 6 [The exponential sequence, cont’d. (Negative time)] The real-valued exponential
sequence ( ) , defined for negative is another basic building block in discrete-time systems.
(The minus sign in front of is chosen for a reason.) Here again there are two broad categories: | |< 1
and | |> 1. In this case, however, if is a positive fraction then, as , the sequence ( )
increases monotonically. If is a negative fraction then, as , the sequence ( ) alternates in
sign while increasing in magnitude.
In the MATLAB plots below note that the vertical axis ( ) is at the right edge, not the left.
%Plot the function xn = -b^n
b = 0.9; n = -20: -1; xn = -b.^n;
subplot(2,1,1); stem(n, xn, 'b'); xlabel ('n'), ylabel('x(n)');
title ('x(n) = -b^n for n = -20 to -1'); legend ('x(n) = -(0.9)^n u(-n-1)'); grid;
%
b = -0.9; n = -20: -1; xn = -b.^n;
subplot(2,1,2); stem(n, xn, 'b'); xlabel ('n'), ylabel('x(n)');
title ('x(n) = -b^n for n = -20 to -1'); legend ('x(n) = -(-0.9)^n u(-n-1)'); grid;
-20 -18 -16 -14 -12 -10 -8 -6 -4 -2 0
-10
-5
0
n
x(n)
x(n) = -bn
for n = -20 to -1
x(n) = -(0.9)n
u(-n-1)
-20 -18 -16 -14 -12 -10 -8 -6 -4 -2 0
-10
-5
0
5
10
n
x(n)
x(n) = -bn
for n = -20 to -1
x(n) = -(-0.9)n
u(-n-1)
www.jntuworld.com
www.jntuworld.com
S&S-8 (The z-transform) 15 of 49 Dr. Ravi Billa
If | |> 1 then, as , the sequence ( ) decreases monotonically if is positive.
However, if is negative then, as , the sequence ( ) decreases in magnitude while
alternating in sign. Again, in the MATLAB plots the vertical axis ( ) is at the right edge.
%Plot the function xn = -b^n
b = 1.1; n = -20: -1; xn = -b.^n;
subplot(2,1,1); stem(n, xn, 'b'); xlabel ('n'), ylabel('x(n)');
title ('x(n) = -b^n for n = -20 to -1'); legend ('x(n) = -(1.1)^n u(-n-1)'); grid;
%
b = -1.1; n = -20: -1; xn = -b.^n;
subplot(2,1,2); stem(n, xn, 'b'); xlabel ('n'), ylabel('x(n)');
title ('x(n) = -b^n for n = -20 to -1'); legend ('x(n) = -(-1.1)^n u(-n-1)'); grid;
-20 -18 -16 -14 -12 -10 -8 -6 -4 -2 0
-1
-0.5
0
n
x(n)
x(n) = -bn
for n = -20 to -1
x(n) = -(1.1)n
u(-n-1)
-20 -18 -16 -14 -12 -10 -8 -6 -4 -2 0
-1
-0.5
0
0.5
1
n
x(n)
x(n) = -bn
for n = -20 to -1
x(n) = -(-1.1)n
u(-n-1)
www.jntuworld.com
www.jntuworld.com
S&S-8 (The z-transform) 16 of 49 Dr. Ravi Billa
Example 7 [The sinusoidal sequence] Consider the continuous-time sinusoid x(t)
( )
and are the analog frequency in Hertz (or cycles per second) and radians per second, respectively.
The sampled version is given by
( )
We may drop the from ( ) and write
( )
We may write which is the digital frequency in radians (per sample), so that
( )
Setting gives ( ) which is the digital frequency in cycles per sample. In the
analog domain the horizontal axis is calibrated in seconds; “second” is one unit of the independent
variable, so and are in “per second”. In the digital domain the horizontal axis is calibrated in
samples; “sample” is one unit of the independent variable, so and are in “per sample”.
Periodic signal The discrete-time signal x(n) is periodic if, for some integer N > 0
( ) ( )
The smallest value of N that satisfies this relation is the (fundamental) period of the signal. If there is
no such integer N, then ( )is an aperiodic signal.
Example Given that the continuous-time signal ( ) is periodic, that is, ( ) ( ) for all t, and
that the discrete-time signal ( ) is obtained by sampling ( ) at T-second intervals, under what
conditions will ( ) be periodic?
Solution If ⁄ ⁄ for integers and then ( ) has exactly N samples in L periods of
( ) and ( ) is periodic with period N. If ⁄ for integer , then
( ) ( ) for all , or
( ) ( ) (( ⁄̅̅̅̅̅̅̅) ) (( ) ) for all
which implies periodicity with a period of .
More generally, if ⁄ ⁄ for integers and , then
( ) ( ) for all , or
( ) ( ) (( ⁄̅̅̅̅̅̅̅̅̅) ) (( ) ) for all
which again implies a period of . Thus ( ) will be periodic if is a rational number but not
otherwise.
Periodicity of sinusoidal sequences The sinusoidal sequence ( ) has several major differences
from the continuous-time sinusoid as follows:
a) The sinusoid ( ) ( ) ( ) is periodic if , that is, , is rational. If is
not rational the sequence is not periodic. Replacing n with (n+N) we get
www.jntuworld.com
www.jntuworld.com
S&S-8 (The z-transform) 17 of 49 Dr. Ravi Billa
( )= ( ( ))= ( ) ( )+ ( ) ( )
Clearly ( ) will be equal to ( ) – that is, the above expression reduces to ( ) – if
, an integer or . The fundamental period is obtained by choosing m as the smallest
integer that yields an integer value for N. For example, if = 15/25, which in reduced fraction form is
3/5, then we can choose m = 3 and get N = 5 as the (fundamental) period. In general, if is rational
then = where p and q are integers. If is in reduced fraction form then the denominat or q is
the period as in the above example.
On the other hand if is irrational, say √ , then N will not be an integer, and thus x(n) is
aperiodic.
b) The sinusoidal sequences ( ) and (( ) ) for 0 ω0 2π are identical. This can be
shown using the identity
(( ) ) ( )
= ( ) ( ) ( ) ( ) QED
Similarly, ( ) and (( ) ) are the same. Therefore in considering sinusoidal
sequences for analysis purposes can be restricted to the range without any loss of
generality.
Example 8 [The complex exponential sequence] Obtain the discrete-time sequence corresponding to
( ) . The signal ( ) may be discretized by replacing t with nT,
yielding the sequence
( )
Setting we have
( )
The sequence or is periodic if ( ) is rational. The parameters and
=( ) are the digital frequency in radians/sample and cycles/sample, respectively.
Note: In expressions such as ( ) or ( ) or or we shall loosely refer to ω or f as
the (digital) frequency even when the signal concerned is not periodic by the definition of periodicity
given above.
Example9 Plot the sequences ( ) and ( ) ( ). What are their
“frequencies”? Which of them is truly periodic and what is its periodicity?
Solution The MATLAB program segment follows:
N = 21; n = 0: N-1;
%
%Nonperiodic
x1= 2*cos(1*n);
subplot(2, 1, 1), stem(n, x1);
xlabel('n'), ylabel('x1'); title('x1 = 2 cos 1n');
%
www.jntuworld.com
www.jntuworld.com
S&S-8 (The z-transform) 18 of 49 Dr. Ravi Billa
%Periodic
x2 = 2*cos(0.2*pi*n);
subplot(2, 1, 2), stem(n, x2);
xlabel('n'), ylabel('x2'); title('x2 = 2 cos 0.2pi n');
0 2 4 6 8 10 12 14 16 18 20
-2
-1
0
1
2
n
x1
x1 = 2 cos 1n
0 2 4 6 8 10 12 14 16 18 20
-2
-1
0
1
2
n
x2
x2 = 2 cos 0.2 n
www.jntuworld.com
www.jntuworld.com
S&S-8 (The z-transform) 19 of 49 Dr. Ravi Billa
Example10 Plot the sequence ( ) ( ) ( ).
Solution The MATLAB program segment follows:
n = [0: 30];
%
%“.^” stands for element-by-element exponentiation
%“.*” stands for element-by-element multiplication
x3 = 2* ((0.9) .^n) .*cos(0.2*pi*n);
stem(n, x3);
xlabel('n'), ylabel('x3'); title('Sequence x3(n)');
The sum of two discrete-time periodic sequences is also periodic. Let x(n) be the sum of two periodic
sequences, x1(n) and x2(n), with periods N1 and N2 respectively. Let p and q be two integers such that
= and
(p and q can always be found)
Then x(n) is periodic with period N since, for all n,
( ) ( ) ( )
( ) ( )
( ) ( )
( )
Definition A discrete-time signal is a sequence, that is, a function defined on the positive and negative
integers.
0 5 10 15 20 25 30
-1.5
-1
-0.5
0
0.5
1
1.5
2
n
x3
Sequence x3(n)
www.jntuworld.com
www.jntuworld.com
S&S-8 (The z-transform) 20 of 49 Dr. Ravi Billa
The sequence ( ) ( ) ( ) is a complex (valued) sequence if ( ) is not zero for all
n. Otherwise, it is a real (valued) sequence.
Signal descriptions such as x1(n) = 2 cos 3n or x2(n) = 3 sin (0.2πn) are an example of signal
representation in functional form. Alternatively, if a signal is non-zero over a finite (small enough)
interval, we can list the values of the signal as the elements of a sequence. For example
x3(n) = {5, 2, – 1, 1, – 1/2, 4}

The arrow indicates the value at n = 0. We omit the arrow when the first entry represents the value for n
= 0. The above sequence is a finite length sequence. It is assumed that all values of the signal not listed
are zero. In the above example x(0) = 1, x(1) = –1/2, x(–4) = x(3) = 0, etc.
Definition A discrete-time signal whose values are from a finite set is called a digital signal.
Odd and even sequences The signal x(n) is an even sequence if ( ) ( ) for all n, and is an odd
sequence if ( ) ( ) for all n.
The even part of ( ) is determined as
( )
( ) ( )
and the odd part of ( ) is given by
( )
( ) ( )
The signal ( ) then is given by ( ) ( ) ( ).
Definition A discrete-time sequence ( ) is called causal if it has zero values for n< 0, i.e., ( )= 0
for n< 0. If this condition is not satisfied the signal is non-causal. (An anticausal sequence has zero
values for , i.e., ( )= 0 for .)
x(n) (Even)
n
0 1
2
–1
–2 –1
x(n) (Odd)
n
0
2
–2 1
www.jntuworld.com
www.jntuworld.com
S&S-8 (The z-transform) 21 of 49 Dr. Ravi Billa
The unit sample sequence (discrete-time impulse, aka the Kronecker delta)
( )
Whereas δ(n) is somewhat similar to the continuous-time impulse function δ(t) – the Dirac delta
– we note that the magnitude of the discrete impulse is finite. Thus there are no analytical difficulties in
defining δ(n). It is convenient to interpret the delta function as follows:
( )
The unit step sequence
( )
( )
a) The discrete delta function can be expressed as the first difference of the unit step function:
( ) ( ) ( )
b) The sum from – to n of the δ function gives the unit-step:
∑ ( ) ( )
δ(n)
n
0 1–1
1 1
δ(n–k)
n
0 1–1 k
www.jntuworld.com
www.jntuworld.com
S&S-8 (The z-transform) 22 of 49 Dr. Ravi Billa
Results (a) and (b) are like the continuous-time derivative and integral respectively.
c) By inspection of the graph of u(n), shown below, we can write:
( ) ( ) ( ) ( ) ∑ ( )
d) For any arbitrary sequence x(n), we have
( ) ( ) ( ) ( )
that is, the multiplication will pick out just the one value x(k).
If we find the infinite sum of the above we get the sifting property:
∑ ( ) ( ) ( )
e) We can write x(n) as follows:
( ) ( ) ( ) ( ) ( ) ( ) ( ) ( ) ( )
This can be verified to be true for all n by setting in turn
The above can be written compactly as
n
Sum up to here is zero
k
0
k
0 n
Sum up to here is 1
0
δ(n)
u(n)
n
δ(n–1)
1
δ(n–2)
2
δ(n–3)
3
www.jntuworld.com
www.jntuworld.com
S&S-8 (The z-transform) 23 of 49 Dr. Ravi Billa
( ) ∑ ( ) ( )
This is a weighted-sum of delayed unit sample functions.
8.3 The z-transform
For continuous-time systems the Laplace transform is an extension of the Fourier transform. The
Laplace transform can be applied to a broader class of signals than the Fourier transform can, since
there are many signals for which the Fourier transform does not converge but the Laplace transform
does. The Laplace transform allows us, for example, to perform transform analysis of unstable systems
and to develop additional insights and tools for linear time-invariant (LTI) system analysis.
The z-transform is the discrete-time counterpart of the Laplace transform. The z-transform
enables us to analyze certain discrete-time signals that do not have a discrete-time Fourier transform.
The motivations and properties of the z-transform closely resemble those of the Laplace transform.
However, as with the relationship of the continuous time versus the discrete-time Fourier transforms,
there are distinctions between the Laplace transform and the z-transform.
Definition The two-sided (bilateral) z-transform, X(z), of the sequence x(n) is defined as
( ) { ( )} ∑ ( )
where is the complex variable. The above power series is a Laurent series.
The one–sided (unilateral) z-transform is defined as
( ) ∑ ( )
The unilateral z-transform is particularly useful in analyzing causal systems specified by linear
constant-coefficient difference equations with nonzero initial conditions into which inputs are stepped.
It is extensively used in digital control systems.
The region of convergence (ROC) is the set of z values for which the above summation converges. In
general the ROC is an annular region in the complex z-plane given by
| |
Im
Re
ROC
Rx–
Rx+
z-plane
www.jntuworld.com
www.jntuworld.com
S&S-8 (The z-transform) 24 of 49 Dr. Ravi Billa
Relationship between the z-transform and the discrete-time Fourier transform In the continuous-
time situation we denote the Laplace transform of ( ) by ( ) where = is the transform
variable in the complex -plane. The variable is the analog frequency in radians per second. The
corresponding Fourier transform is denoted by ( ). When the Laplace transform reduces to
the Fourier transform. In other words, when evaluated on the imaginary axis (i.e., for = ) in the -
plane the Laplace transform reduces to the Fourier transform.
There is a similar relationship between the z-transform of the sequence ( ) and its discrete-
time Fourier transform. To explore this we express the complex variable z in polar form as
where and are respectively the magnitude and angle of . The variable ω is the digital frequency in
radians per sample. Substituting for in the definition of the -transform gives us
( )| ∑ ( )( ) ∑ [ ( )]
Equivalently,
( ) ∑ [ ( )]
In other words, ( ) is the Fourier transform of the sequence ( ) multiplied by the real
exponential (convergence factor) . The exponential weighting may decay or grow with
increasing , depending on whether is greater than or less than 1. In particular, for | | the -
transform reduces to the discrete-time Fourier transform, that is,
( )| ( ) ∑ ( )
Thus, the z-transform, when evaluated on the unit circle, reduces to the discrete-time Fourier transform.
In the discussion of the -transform the role of the unit circle in the -planeis similar to that of the
imaginary axis in the -planein the discussion of the Laplace transform.
The -transform of ( ) then is the Fourier transform of ( ) :
( ) ( ) { ( ) } ∑ [ ( )]
From this equation, it is seen that, for convergence of the -transform, we require that the Fourier
transform of ( ) converge. For any specific ( ), we would expect this convergence for some
values of and not for others. In general, the -transform of a sequence, ( ), has a region of
convergence (ROC) – the range of values of for which ( ) converges. If the ROC includes the unit
circle, then the Fourier transform of ( ) also converges.
jΩ
σ
s-plane
1
Im
Re
z-plane
www.jntuworld.com
www.jntuworld.com
S&S-8 (The z-transform) 25 of 49 Dr. Ravi Billa
8.4 Transforms of some useful sequences
Example 1 [Exponential ( )] The positive-time signal
( )
If a is a positive fraction this sequence decays exponentially to 0 as n → ∞. Substituting x(n) into the
defining equation, the z-transform is
{ ( )} ( ) ∑ ∑( ) | |
| | | |
The ROC is | | | |. This X(z) is a rational function (a ratio of polynomials in z). The roots of
the numerator polynomial are the zeros of X(z) and the roots of the denominator polynomial are the
poles of X(z). If | | , the ROC does not include the unit circle; for such values of , the Fourier
transform of ( ) does not converge.
This is a right-sided sequence. A right-sided sequence has an ROC that is the exterior of a circle
with radius Rx– (| | | |in this case, so the radius is | |). If the ROC is the exterior of a circle the
underlying sequence is a right-sided sequence.
Definition A right-sided sequence ( ) is one for which ( ) for all n < n0 where n0 is positive
or negative but finite. If n0 0 then x(n) is a causal or positive-time sequence.
Example 2 [Exponential ( ) (negative time)] Recall that the unit step sequence u(.) = 1 if
the argument of u(.) is  0, i.e., if (–n–1)  0 or n  –1.
( )
If b> 1 this sequence decays exponentially to 0 as n → –∞. The z-transform is,
Im
Re
a
Pole at a
Zero at 0
ROC, |z| > |a|
(Shaded area)
z-plane
www.jntuworld.com
www.jntuworld.com
S&S-8 (The z-transform) 26 of 49 Dr. Ravi Billa
{ ( )} ( ) ∑ ( ) ∑ ∑ ( )
Let n = –m and change the limits accordingly to get,
( ) ∑ ( ) ∑( )
We added 1 in the last step above to make up for the m = 0 term within the summation. The result is,
( )
( )
| |
| | | |
This is a left-sided sequence. A left-sided sequence has an ROC that is the interior of a circle,
z < Rx+. In this case the ROC is z < b .If the ROC is the interior of a circle the underlying sequence is
a left-sided sequence.
Definition A left-sided sequence ( ) is one for which ( ) for all n  n0, where n0 is positive or
negative but finite. If n0 0 then x(n) is an anticausal or negative-time sequence.
Note that if b = a then the two examples above have exactly the same X(z). So what makes the
difference? The region of convergence makes the difference.
Example 3 [Two-sided sequence] This is the sum of the positive- and negative-time sequences of the
previous two examples.
( ) ( ) ( )
Substituting into the defining equation,
( ) { ( )} ∑ [ ( ) ( )] ∑ ∑
Now, from Examples 1 and 2,
∑ | | | | ∑ | | | |
So, the desired transform Y(z) has a region of convergence equal to the intersection of the two separate
ROC’s | | | |and | | | |. Thus
b
Im
Re
zero
pole
z-plane
ROC
www.jntuworld.com
www.jntuworld.com
S&S-8 (The z-transform) 27 of 49 Dr. Ravi Billa
( ) (| | | |) (| | | |)
( )
( )( )
| | | | | |
The ROC is the overlap of the shaded regions, that is, the annular region between | |and| |. The two
zeros are at 0 and( ) , and the two poles at a and b.
If b < a the transform does not converge.
In the above three examples we may express the z-transform both as a ratio of polynomials in z
(i.e., positive powers) and as a ratio of polynomials in z–1
(negative powers). From the definition of the
z-transform, we see that for sequences which are zero for n< 0, X(z) involves only negative powers of z.
However, reference to the poles and zeros is always in terms of the roots of the numerator and
Im
Re
a b
poles
zero zero(a+b)/2For |a| < |b|
ROC
Im
Re
b a
poles
zero
zero
No ROC
when |b| < |a|
|z| > |a|
www.jntuworld.com
www.jntuworld.com
S&S-8 (The z-transform) 28 of 49 Dr. Ravi Billa
denominator expressed as polynomials in z. Also, it is sometimes convenient to refer to X(z), written as
a ratio of polynomials in z (i.e., positive power of z), as having poles at infinity if the degree of the
numerator exceeds the degree of the denominator or zeros at infinity if the numerator is of smaller
degree than the denominator.
Example 4 [Finite-length sequence] Only a finite number of sequence values are non-zero, as given
below.
( )
By the defining equation we have
( ) ∑ ( ) ( ) ( )
Convergence of this expression requires simply that ( ) for . Then z may take on all
values except z =  if N1 is negative and z = 0 if N2 is positive. Thus the ROC is at least 0 < z < and it
may include either z = 0 or z =  depending on the sign of N1 and N2.
Example 5 [Unit sample ( )]
{ ( )} ∑ ( )
Delayed unit sample ( )
{ ( )} ∑ ( ) | |
| | | |
Example 6 [Unit step ( )]
{ ( )} ∑ ( ) ∑
| | | |
Example 7 [Unit step ( )(negative time)]
{ ( )} ∑ ( ) ∑ ( ) ∑
∑ ( )
| |
8.5 Important properties of z-transforms
The proofs are easily obtained by using the definition of the z-transform and transformations in the
summation.
www.jntuworld.com
www.jntuworld.com
S&S-8 (The z-transform) 29 of 49 Dr. Ravi Billa
(1) Linearity If { ( )} ( ) with { | | } and { ( )} ( ) with
{ | | }, then { ( ) ( )} ( ) ( ) with ROC at least the overlap of the
ROC’s of ( ) and ( ). If there is any pole-zero cancellation due to the linear combination, then the
ROC may be larger.
(2) Translation (Time-shifting) If { ( )} ( ) with { | | } then { ( )}
( ) with the same ROC except for the possible addition or deletion of z = 0 or z = ∞ due to .
Example 1 Given x(n) = {1, 2} and x2(n) =x(n+2) find X(z) and X2(z) and their respective
ROCs. ( ) , ROC: entire z-plane except z = 0; ( ) ( ) , ROC:
entire z-plane except z = ∞.
(3) Multiplication by a complex exponential sequence (Scaling in the z-domain) If { ( )} ( )
with { | | } then { ( )} ( )| ( ) with {| | | | | | }.
Example 2 Given ( )= {1, 2} and ( ) ( ) ( ) find X(z) and X2(z) and their respective
ROCs.
(4) Multiplication by a ramp (Differentiation in the z-domain) If { ( )} ( ) with ,
then
{ ( )}
( )
Example 3 Given ( )= {1, 2} and ( ) ( ) ( ) find ( )and ( ) and their
respective ROCs.
{ ( )} {( ) ( )} { ( )} { ( )} { [ ( )]}
(5) Time reversal If { ( )} ( )with { | | } then { ( )} ( ) with
{( ) | | ( )}.
Example 4 Given ( ) ( ) and ( ) ( ) determine ( ).
( ) ( ) ( ), ( ) , ROC: 0.5 <|z|
ʓ-1{ )( 1
zX } = x(–n); x2(n) = ʓ-1{ )( 1
zXz }= x(–(n+1)).= ))1((2 ))1((

nun
(6) Convolution in time domain leads to multiplication in frequency domain Given { ( )}
( ) with { }and { ( )} ( ) with { } and
( ) ( ) ∑ ( ) ( )
then
{ ( ) ( )} ( ) ( ) { }
(7) Multiplication in time domain leads to convolution in frequency domain If { ( )} ( )
with { | | } and { ( )} ( ) with { | | } then
{ ( ) ( )} ∮ ( ) ( )
www.jntuworld.com
www.jntuworld.com
S&S-8 (The z-transform) 30 of 49 Dr. Ravi Billa
with { | | } where is a complex contour integral and is a closed contour in
the intersection of the ROCs of ( ) and ( ).
(8) Initial Value Theorem If x(n) is a causal sequence with z transform X(z), then
x(0)= )(lim zX
z 
(9) Final Value Theorem If { ( )} ( ) and the poles of ( ) ( ) are all inside the unit circle
then the value of x(n) as n→ is given by
( )| [( ) ( )]
Some also give this as
( )| [( ) ( )]
Example Can the final value theorem be applied to the following z-transform?
( )
( )( )
Solution For the final value theorem to be applicable ( ) ( ) must not have any poles on
oroutside the unit circle. Here
( ) ( )
( ) ( )( )
Since there is a pole at , the final value theorem does not apply. Because of the pole at ,
( ) is unbounded as .
8.6 Transforms of some useful sequences, cont’d.
Example 1 [Unit ramp ( )] If
{ ( )} ( )
then
{ ( )}
( )
( )
{( ) }
( )
( )
| | ( )
Example 2 [Sinusoid ( ) ( )]
{ ( )} ∑ ( ) ∑ ( )
(∑( ) ∑( ) )
( ) | | | |
( )
(
( ) ( )
( )
)
(
( )
( )
)
www.jntuworld.com
www.jntuworld.com
S&S-8 (The z-transform) 31 of 49 Dr. Ravi Billa
Using the identities
we have
{ ( )} | |
Example 3 [ ( ) ( )] As an extension of the above example and using property #3
{ ( )} |
( )
( )
( ) ( )
| |
Alternative: We may use the defining equation
{ ( )} ∑ ( ) ∑ ( ) ( )
Example 4 [Cosinusoid ( ) ( )] Using the relation ( )⁄ and a
procedure similar to that for the sinusoid we get
{ ( )} ∑ ( ) ∑ ( )
…
{ ( )} | |
Example 5 [ ( ) ( )] As an extension of the above example and using property #3,
{ ( )} |
( )
( ⁄ ) ( ⁄ )
( ⁄ ) ( ⁄ )
| |
Alternative: We may use the defining equation
{ ( )} ∑ ( ) ∑ ( ) ( )
www.jntuworld.com
www.jntuworld.com
S&S-8 (The z-transform) 32 of 49 Dr. Ravi Billa
Common z-transform pairs
# Sequence z-transform ROC
1 ( ) 1 Entire z-plane
2 ( ) | |
3 ( ) | |
4 ( ) | |
5 ( ) | |
6 ( )
( )
| |
7 ( ) ( ) | |
8 ( ) ( ) | |
9 ( ) ( ) | |
10 ( ) ( ) | |
8.7 Region of convergence (ROC)
Given a discrete-time sequence ( ) which may be a signal or an impulse response we denote its z-
transform as ( ). The transform will be a rational function whenever ( ) is a linear combination of
real or complex exponentials. In other words, it may be expressed as either a ratio of polynomials in
or as a ratio of polynomials in . It is characterized by a unique region of convergence.
When the z-transform is expressed as a ratio of polynomials in we define its poles and zerosin
terms of the roots of the denominator and numerator, respectively. Also, it is sometimes convenient to
refer to the transform as having poles at infinity if the degree of the numerator exceeds that of the
denominator or zeros at infinity if the numerator is of lesser degree than the denominator.
Example 1 [Pole-zero plot and ROC] Give the pole-zero plot for
( )
Assuming that ( ) is causal indicate the ROC of ( ).
Solution The denominator has roots (poles) at
√ ( ) √( ) ( )( )
( )
√ √
There is a zero at z = 0. Further, since the denominator degree is greater than the numerator degree by 1
it is clear that ( )= 0, so that there is an additional zero at z = ∞.
www.jntuworld.com
www.jntuworld.com
S&S-8 (The z-transform) 33 of 49 Dr. Ravi Billa
In MATLAB the transfer function is specified as a ratio of polynomials in 1
z
( )
The numerator coefficients, { }and the denominator coefficients { }are
specified as the two vectors b = [0, 1] and a = [1, -1, -1].
%Pole-zero plot
b = [0, 1]; a = [1, -1, -1]; zplane (b, a)
In the pole-zero plot, a pole is marked by an x and a zero by a little circle.
With
( )
( )( )
we have
( )
and the ROC = {| | } {| | } = {| | }.
Example 2 [Pole-zero plot and ROC] Give the pole-zero plot for
( )
Assuming that ( ) is causal indicate the ROC of ( ).
Solution From
-1 -0.5 0 0.5 1 1.5
-1
-0.8
-0.6
-0.4
-0.2
0
0.2
0.4
0.6
0.8
1
Real Part
ImaginaryPart
www.jntuworld.com
www.jntuworld.com
S&S-8 (The z-transform) 34 of 49 Dr. Ravi Billa
( )
we can see that there are 9 poles at z = 0 and 8 zeros at sundry places and an additional zero at z = ∞
owing to the denominator degree being greater than the numerator degree by 1.
For the MATLAB segment the numerator and denominator coefficients are taken from
( )
%Pole-zero plot
b = [0: 9]; a = [1, 0]; zplane (b, a)
In the pole-zero plot, the pole at is marked by an x with a 9 alongside it indicating its multiplicity.
The zeros are indicated by little circles.
Since ( ) contains only negative powers of the ROC excludes = 0 and is given by ROC =
{| | }
Properties of the ROC
The impulse response and its z-transform are usually denoted by ( ) and ( ), respectively.
However, in what follows we use the symbols ( ) and ( ) to represent any signal including input,
output and impulse response.
(1) If ( ) is a right-sided sequence, the ROC of ( ) is the exterior of a circle centered at the origin
with radius equal to the largest (in magnitude) of the poles of ( ). All other poles are inside this circle.
-1.5 -1 -0.5 0 0.5 1 1.5
-1
-0.5
0
0.5
1
9
Real Part
ImaginaryPart
www.jntuworld.com
www.jntuworld.com
S&S-8 (The z-transform) 35 of 49 Dr. Ravi Billa
(2) If ( ) is a left-sided sequence, the ROC of ( ) is the interior of a circle centered at the origin
with radius equal to the smallest (in magnitude) of the poles of ( ).All other poles are outside this
circle.
(3) If ( ) is a two-sided sequence, the ROC of ( ), if it exists, is an annular ring centered at the
origin.The inner boundary of the ROC is a circle of radius equal to the largest (in magnitude) of the
poles of ( ) arising from the right-sided sequences that make up ( ); and the outer boundary is a
circle of radius equal to the smallest (in magnitude) of the poles of ( ) arising from the left-sided
sequences that make up ( ).
(4) If ( ) is a finite-length sequence, the ROC of ( ) is the entire -plane, except possibly = 0
and/or = ∞. = 0 is excluded if there is a non-zero ( ) value for ; = ∞ is excluded if there is
a non-zero ( ) value for .
(5) The ROC does not contain any poles.
8.8 Inverse z-transform by partial fractions
(Aside) Comparison of inverse z-transform methods A limitation of the power series method is that
it does not lead to a closed form solution (although this can be deduced in simple cases), but it is simple
and lends itself to computer implementation. However, because of its recursive nature care should be
taken to minimize possible build-up of numerical errors when the number of data points in the inverse
z-transform is large, for example by using double precision.
Both the partial fraction method and the inversion integral method require the evaluation of
residues albeit performed in different ways. The partial fraction method requires the evaluation of the
residues of ( )or { ( ) }. The complex inversion integral requires the evaluation of the residues of
( ) . In many instances evaluation of the complex inversion integral is needlessly difficult and
involved.
Both the partial fraction method and the inversion integral method lead to closed form solutions.
The main disadvantage is having to factorize the denominator polynomial of X(z) when it is of order
greater than 2. Another disadvantage is multiple order poles and the resulting differentiation(s) when
determining residues.
The partial fraction method directly generates the coefficients of parallel structures for digital
filters. The inversion integral method is widely used in the analysis of quantization errors in discrete-
time systems.
(End of Aside)
As in Laplace transforms, in order to expand a rational function into partial fractions, the degree
of the numerator should be less than the degree of the denominator – proper fraction. If it is not then we
perform long division as below. As a result ( )is the quotient and ( )is the remainder with degree
less than that of ( ).
( )
( )
( )
( )
( )
( )
Long Division
Q(z) ←Quotient
Denominator→ D(z) N(z) ←Numerator
---
N1(z) ←Remainder
www.jntuworld.com
www.jntuworld.com
S&S-8 (The z-transform) 36 of 49 Dr. Ravi Billa
The long division is done until we get a remainder polynomial ( )whose degree is less than the
degree of the denominator ( ). We then obtain x(n) as
( ) { ( )} { ( )} {
( )
( )
}
Since ( ) ( )⁄ is a proper fraction it can be expanded into partial fractions. The overall inverse
transform is obtained by looking up a table of z-transform pairs.
However, there is an alternative available in the case of z-transforms which is not available in
Laplace transforms. This is a result of the fact that z-transforms are characterized by a z in the
numerator (as can be verified by looking at a table of z-transforms). Therefore, instead of expanding
( )we may, instead, expand [ ( )⁄ ] into partial fractions giving
( )
so that ( )is given by
( )
This can be inverted by a simple look-up of a table of transforms. Note also that in some cases
( ) ( ) ( ) may not be a proper fraction but [ ( )⁄ ] is and, therefore, this method obviates
the need for long division of ( ) by ( ). (In still other cases even [ ( )⁄ ] may not be a proper
fraction. See later under “General procedure for partial fraction expansion”.)
Example 1 (See also long division later). Find the inverse z-transform, using partial fractions, of
( )
( )
( )
State your assumptions.
Solution This is not a proper fraction since the degree of the numerator is not less than the degree of the
denominator. However, [ ( )⁄ ] is a proper fraction.
( )
( )( )
whose partial fraction expansion is obtained as below:
( )
( )( )
( )
|
( )
( )
( )
|
( )
( )
( )
( )
By looking up a table of z-transforms the inverse z-transform is
{ ( )} ( ) ( ) ( )
Note that we are giving here the causal solution that corresponds to a ROC |z| > 2 (not 1 < |z| < 2 or |z|
< 1) so that x(n) is a right-sided sequence.
www.jntuworld.com
www.jntuworld.com
S&S-8 (The z-transform) 37 of 49 Dr. Ravi Billa
The alternative method is to divide ( ) by ( )as below (as is standard practice in Laplace
transforms). Note that in this long division the numerator and denominator polynomials are arranged in
the order of decreasing powers of z. There are three other ways (all of them wrong) of arranging the two
polynomials for the long division.
Long Division
2 ←Quotient
Denominator→ z2
–3z + 2 2z2
– 3z ←Numerator
2z2
– 6z + 4
3z – 4 ←Remainder
Thus ( ) can be expressed as
( )
( )( )
( )
where
( )
( )( )
is a proper fraction and can be expanded into partial fractions as below:
( )
( )( ) ( ) ( )
Solving for A and B we get A = 1 and B = 2, so that X(z) may be written
( )
( ) ( )
Taking the inverse z-transform we get
( ) { ( )} {
( ) ( )
}
{ } { } { }
A term like { } is handled by writing ( )as ( ). We know that { } ( ), so
{ } { } ( )
Similarly
{ } { } ( )
Thus
( ) ( ) ( ) ( )
This can be verified to be equivalent to ( ) ( ) ( ) obtained earlier.
Example 2 Find the inverse transform of
( )
where the ROC is (a) |z|> 1, (b) |z|< (1/3), (c) (1/3) <|z|< 1.
Solution The three possible regions of convergence are shown below. The example illustrates that the
inverse transform, x(n), is unique only when the ROC is specified.
www.jntuworld.com
www.jntuworld.com
S&S-8 (The z-transform) 38 of 49 Dr. Ravi Billa
(a) For ROC ≡ |z|> 1
( )
( ) ( ) ( ( ) ( ))
( )( ( ⁄ )) ( ⁄ )
( ( ⁄ ))
|
and
( )
|
( ) ( ⁄ ) ( ⁄ )
( ⁄ )
( )
( ⁄ ) ( ⁄ )
( ⁄ )
The inverse is
( ) { ( )} {
( ⁄ ) ( ⁄ )
( ⁄ )
}
{
( ⁄ )
} {
( ⁄ )
( ⁄ )
}
The ROC is outside the largest pole signifying a right-sided sequence for each pole. The inverse
becomes
( ) ( ⁄ )( ) ( ) ( ⁄ )( ⁄ ) ( ) {( ⁄ ) ( ⁄ )( ⁄ ) } ( )
(b) For ROC ≡ |z|< (1/3). The partial fraction expansion does not change. Since the ROC is inward of
the smallest pole, x(n) consists of two negative-time sequences.
( ) { ( )} {
( ⁄ ) ( ⁄ )
( ⁄ )
}
{
( ⁄ )
} {
( ⁄ )
( ⁄ )
}
( ) ( ) ( ( ) ) ( )
{ ( ) } ( )
( ⁄ ){ ( ⁄ ) } ( )
(a) (b) (c)
www.jntuworld.com
www.jntuworld.com
S&S-8 (The z-transform) 39 of 49 Dr. Ravi Billa
(c) For ROC ≡ (1/3) <|z|< 1. The partial fraction expansion stays the same. The pole at z = 1
corresponds to a negative-time sequence (left-sided sequence) while the pole at z = 1/3 gives a positive-
time sequence (right-sided sequence). This is the only possibility.
( ) { ( )} {
( ⁄ ) ( ⁄ )
( ⁄ )
}
{
( ⁄ )
} {
( ⁄ )
( ⁄ )
}
( ) ( ) ( ) ( )
( ) ( ) ( )
The overall result is a two-sided sequence.
Example 3 (Sometimes there is no z in the numerator to factor out, but we still divide X(z) by z as in
this example.) Find ( )for
( ) | |
Solution
( )
( ) ( )( ( ⁄ )) ( ⁄ )
( )( ( ⁄ ))
|
( )( ⁄ )
( ( ⁄ ))
|
( )
|
( )
( ⁄ )
( )
( ⁄ )
( ) { ( )} {
( ⁄ )
}
Re
Im
1
(c) ⁄ | |
1/3
www.jntuworld.com
www.jntuworld.com
S&S-8 (The z-transform) 40 of 49 Dr. Ravi Billa
( ) ( ) ( ) ( )
Example 4 Find the inverse z-transform of
( )
( )
Assume that ( ) is causal.
Solution The roots of the quadratic in the denominator are given by
√ ( ) √( ) ( )( )
( )
√ √
( )
( )
( )( )
( ) ( )
( )( )
( )
( )( )
|
( )
( )
|
( )
( )
|
( ) ( ⁄ )
( )
( ⁄ )
( ) { ( )} {
( ⁄ )
}
( ) ( ) ( ) ( ) ( )
Example 5 Find the inverse z-transform of
( )
( ) ( )
| |
Solution
( )
( ) ( ) ( ⁄ ) ( ⁄ )
( ) ( )
|
( )
|
www.jntuworld.com
www.jntuworld.com
S&S-8 (The z-transform) 41 of 49 Dr. Ravi Billa
( )
|
( )
( ⁄ ) ( ⁄ )
( )
( ⁄ ) ( ⁄ )
( ) ( ) ( ) ( ) ( ) ( )
Example 6 Find the inverse of
( )
( ) ( )
⁄ | |
Solution
( )
( ) ( ) ( ) ( )
There is a pole at z = ∞. The numerator degree is 3 and is greater than the denominator degree. By long
division we reduce the numerator degree by 2 so that the resulting numerator degree is less than that of
the denominator degree.
( )
( ) ( )
(
( ⁄ ) ( ⁄ )
( ⁄ ) ( ⁄ )
)
(Note that in the long division leading to the above result the numerator and denominator polynomials
are arranged in the order of decreasing powers of z. There are three other ways (all of them wrong) of
arranging the two polynomials for the long division.)
The proper fraction part can now be expanded into partial fractions:
( ⁄ ) ( ⁄ )
( ⁄ ) ( ⁄ ) ( ⁄ ) ( ⁄ )
( ⁄ ) ( ⁄ )
( ⁄ )
|
( ⁄ ) ( ⁄ )
( ⁄ )
|
( ) ( )
( ⁄ )
( )
( ⁄ )
( )
( )
( ⁄ )
( )
( ⁄ )
( ) ( ) ( ) { ( ) ( ) } ( )
The answer has values for n = –1 and n = –2 due to the double pole at z = ∞. The resulting x(n) is not a
causal sequence.
Example 7 Assuming that
( )
is a causal system function, prove the following independently of each other.
www.jntuworld.com
www.jntuworld.com
S&S-8 (The z-transform) 42 of 49 Dr. Ravi Billa
(a) ( ) ( ⁄ ) ( ) ( ⁄ ) ( )
(b) ( ) ( ) ( ) ( )
(c) ( ) ( ) ( ) ( )
Solution
(a) Expand [ ( )⁄ ] into partial fractions
( ) ( ⁄ ) ( ⁄ )
(b) Write ( ) ( ) ( )⁄ as
( )
(c) By long division
( )
Inverse z-transform when there are repeated roots With repeated roots, that is, a -th order pole at
we have ( )in the form
( )
( )
| |
The table below gives the inverse z-transforms for several values of k and for the general case of
arbitrary k.
Repeated Roots
( ) ( ) { ( )} | | | |
( )
( )
( )
( )
( )
( )
( )
( )
( )( )
( )
… …
( )
( )( ) ( ̅̅̅̅̅̅̅)
( )
̅̅̅̅̅̅
( )
Example 10 Find the inverse of
( )
( ) ( )
| |
Solution
( )
( ) ( ) ( ) ( ) ( )
( ) ( )
|
( ) ( )
www.jntuworld.com
www.jntuworld.com
S&S-8 (The z-transform) 43 of 49 Dr. Ravi Billa
( )
|
( ) ( )
( )
( )
|
( ) ( )
( )
{ (
( )
)}| { (
( )
)}|
{
( ) ( ) ( ) ( )
( )
}|
( )
( )
( )
( ) ( )
( )
( ) ( ) ( )
( )
( ) ( ) ( )
Taking the inverse z-transform,
( ) { ( )} {
( ) ( ) ( )
}
{ } {
( )
} {
( )
} {
( )
}
( ) ( ) ( ) ( ) ( ) ( ) ( )
Other possibilities If we choose to expand ( ), rather than [ ( ) ], into partial fractions, we need
to perform long division to reduce the degree of the numerator by 1 resulting in
( ) ( )
where ( )is the proper fraction part of the above
( )
( ) { ( )} ( ) { ( )}
Either ( ) itself or [ ( ) ] may be expanded into partial fractions.
Example 11 Find the inverse of
www.jntuworld.com
www.jntuworld.com
S&S-8 (The z-transform) 44 of 49 Dr. Ravi Billa
( )
( ) ( )
| |
Solution
( )
( ) ( ) ( ) ( ) ( ) ( )
( )
|
( )
( )
( )
{ (
( )
)}|
( )
{ (
( )
)}|
( )
|
( )
( )
( )
( ) ( )
( )
( )
( )
( ) ( ) ( ) ( )
( ) { ( )}
{
( ) ( ) ( ) ( )}
{
( )
} {
( )
} {
( )
} {
( )
}
( )
( ) ( ) ( ) ( ) ( ) ( ) ( ) ( )
The u(n) may be factored out etc.
Example 12 [Differentiation] Find the inverse of
( ) ( ) | | | |
Solution This does not involve partial fractions. Rather we use the property dealing with multiplication
by a ramp in the time domain (differentiation in the z-domain). According to this property if
{ ( )} ( ) with , then
{ ( )}
( )
Note that
( )
( )
( )
Here
www.jntuworld.com
www.jntuworld.com
S&S-8 (The z-transform) 45 of 49 Dr. Ravi Billa
( )
{ ( )}
( )
( )
( ) [
( )
( )
]
( )
( )
( )
{ {
( )
}}
The ROC | | | | implies a right-sided sequence, so that
{
( )
} {
( )
}
[ ( )]| ( ) ( )
Thus we have
( )
{ ( )} { ( )}
Therefore
( ) ( )
( ) ( ) ( )
www.jntuworld.com
www.jntuworld.com
S&S-8 (The z-transform) 46 of 49 Dr. Ravi Billa
8.9 Inverse z-transform by power series expansion (long division)
If the z-transform is expressed as a rational function (a ratio of polynomials in or ) we can use
long division (aka synthetic division) to expand it into a power series. If the transform is expressed as an
irrational function we can use the appropriate power series expansion formula (if) available in
mathematical tables such as the CRC Tables. Note that if the transform is expressed as an irrational
function then the partial fraction expansion method of inversion won’t work.
By definition the z-transform of the sequence x(n) is given by
( ) ∑ ( ) ( ) ( ) ( ) ( )
This is a power series (Laurent series). So by long division we obtain the power series expansion of
X(z) and then, by comparison with the power series definition given above, we can identify the
sequence x(n). In particular the coefficient of z–k
is the sequence value x(k).
The method is useful in obtaining a quick look at the first few values of the sequence x(n). This
approach does not assure an analytical solution. The ROC will determine whether the series has
positive or negative exponents. For right-sided sequences the X(z) will be obtained with primarily
negative exponents, while left-sided sequences will have primarily positive exponents. For an annular
ROC, a Laurent expansion would give both positive- and negative-time terms. This last possibility is
illustrated in the example below by taking a little help from partial fractions.
Example 1 Find the inverse transform, by long division, of
( )
( )( )
where the ROC is (a)| | , (b)| | , (c) | | .
Solution (a) ROC is| | . We expect a right-sided sequence, with predominantly negative exponents
of . For the long division arrange numerator and denominator as decreasing powers of and then
divide; or as increasingly negative power of i.e., and then divide.
z-plane
Im
Re
2
ROC, |z| > |2|
(Shaded area)
1
www.jntuworld.com
www.jntuworld.com
S&S-8 (The z-transform) 47 of 49 Dr. Ravi Billa
2 +3z–1
+5z–2
+ 9z–3
+ … ←Q(z)
D(z)→ z2
–3z + 2 2z2
– 3z ←N(z)
2z2
– 6z + 4
3z – 4
3z – 9 + 6z–1
5 – 6z–1
5 – 15z–1
+ 10z–2
9z–1
– 10z–2
9z–1
– 27z–2
+ 18z–3
17z–2
– 18z–3
…
Thus ( ) . By comparison with the defining equation
( ) ( ) ( ) ( ) ( )
we see that the sequence values are
( ) ( ) , or ( ) for n< 0, and
( ) , ( ) , ( ) , etc.
Alternatively, it is also possible to write X(z) as a ratio of polynomials in z–1
( )
Note that the polynomials are written in the order of increasing negative powers of z, that is, z–1
. Long
division gives (the same answer as obtained earlier):
2 +3z–1
+ 5z–2
+ 9z–3
+ … ←Q(z)
D(z)→ 1– 3z–1
+ 2z–2
2 – 3z–1
←N(z)
2 – 6z–1
+ 4z–2
3z–1
– 4z–2
3z–1
– 9z–2
+ 6z–3
5z–2
– 6z–3
…
Solution (b) The ROC is |z|< 1. We expect a left-sided sequence with predominantly positive
exponents of z. For the long division the polynomials are written in the order of increasing powers of z
(or decreasingly negative powers of z, i.e., z–1
).
–(3/2)z – (5/4)z2
– (9/8)z3
– … ←Q(z)
Im
Re
21
ROC
www.jntuworld.com
www.jntuworld.com
S&S-8 (The z-transform) 48 of 49 Dr. Ravi Billa
D(z)→ 2 – 3z + z2
–3z + 2z2
←N(z)
–3z+(9/2)z2
– (3/2)z3
– (5/2)z2
+ (3/2)z3
– (5/2)z2
+ (15/4)z3
–(5/4)z4
– (9/4)z3
+ (5/4)z4
– (9/4)z3
+ (27/8)z4
– (9/8)z5
Thus X(z) = –(3/2)z– (5/4)z2
– (9/8)z3
– … = …– (9/8)z3
– (5/4)z2
– (3/2)z. By comparing with the defining
equation
X(z) = …x(–3)z3
+ x(–2)z2
+ x(–1)z + x(0) + x(1)z–1
+…
we see that the sequence is given by
x(–1) = –3/2, x(–2) = –5/4, x(–3) = –9/8, …etc., and x(n) = 0 for n 0
The other way of long division is shown below:
–(3/2)z – (5/4)z2
– (9/8)z3
– … ←Q(z)
D(z)→ 2z–2
– 3z–1
+ 1 – 3z–1
+ 2 ←N(z)
– 3z–1
+ (9/2) – (3/2)z
…
(Omit) Solution (c) The ROC is 1 <|z|< 2. We expect a two-sided sequence with both positive and
negative exponents of z. Looking at the pole-zero configuration, the pole at z =1 implies a right-sided
sequence while the pole at z = 2 a left-sided sequence. Obviously just a single long division cannot give
both the left-sided and the right-sided sequences simultaneously. We shall obtain the partial fraction
expansion first and then proceed with the long divisions to obtain the sequences separately. These two
sequences are then combined (added) into one sequence to get the solution. Note that once we use
partial fractions the utility of long division is nullified. We do this only to illustrate the method of long
division.
( )
( )( )
Re
Im
ROC
21
www.jntuworld.com
www.jntuworld.com
S&S-8 (The z-transform) 49 of 49 Dr. Ravi Billa
( )
|
( )
( )
( )
|
( )
( )
( )
( ) ( )
( )
( ) ( )
For the term ( )⁄ we have a right-sided sequence given by long division thus:
1 + z –1
+ z –2
+ z –3
+…… ←Q(z)
D(z)→ z – 1 z ←N(z)
z – 1
1
1 – z–1
z–1
z–1
– z–2
z–2
…
The corresponding sequence is
( )
For the term ( )⁄ we have a left sided sequence
–(1/2)z – (1/4)z2
– (1/8)z3
……. ←Q(z)
D(z)→ –2 + z z ←N(z)
z – (1/2)z2
(1/2)z2
(1/2)z2
– (1/4)z3
(1/4)z3
(1/4)z3
– (1/8)z4
(1/8)z4
…
The corresponding sequence is
( )
The complete sequence is then
( ) ( ) ( )
(End of Omit)
www.jntuworld.com
www.jntuworld.com

More Related Content

What's hot

DSP_FOEHU - MATLAB 02 - The Discrete-time Fourier Analysis
DSP_FOEHU - MATLAB 02 - The Discrete-time Fourier AnalysisDSP_FOEHU - MATLAB 02 - The Discrete-time Fourier Analysis
DSP_FOEHU - MATLAB 02 - The Discrete-time Fourier AnalysisAmr E. Mohamed
 
Introduction to Fourier transform and signal analysis
Introduction to Fourier transform and signal analysisIntroduction to Fourier transform and signal analysis
Introduction to Fourier transform and signal analysis宗翰 謝
 
Ff tand matlab-wanjun huang
Ff tand matlab-wanjun huangFf tand matlab-wanjun huang
Ff tand matlab-wanjun huangSagar Ahir
 
The discrete fourier transform (dsp) 4
The discrete fourier transform  (dsp) 4The discrete fourier transform  (dsp) 4
The discrete fourier transform (dsp) 4HIMANSHU DIWAKAR
 
An Intuitive Approach to Fourier Optics
An Intuitive Approach to Fourier OpticsAn Intuitive Approach to Fourier Optics
An Intuitive Approach to Fourier Opticsjose0055
 
Data Science - Part XVI - Fourier Analysis
Data Science - Part XVI - Fourier AnalysisData Science - Part XVI - Fourier Analysis
Data Science - Part XVI - Fourier AnalysisDerek Kane
 
DSP_FOEHU - Lec 09 - Fast Fourier Transform
DSP_FOEHU - Lec 09 - Fast Fourier TransformDSP_FOEHU - Lec 09 - Fast Fourier Transform
DSP_FOEHU - Lec 09 - Fast Fourier TransformAmr E. Mohamed
 
Dft and its applications
Dft and its applicationsDft and its applications
Dft and its applicationsAgam Goel
 
Dsp 2018 foehu - lec 10 - multi-rate digital signal processing
Dsp 2018 foehu - lec 10 - multi-rate digital signal processingDsp 2018 foehu - lec 10 - multi-rate digital signal processing
Dsp 2018 foehu - lec 10 - multi-rate digital signal processingAmr E. Mohamed
 
Optics Fourier Transform Ii
Optics Fourier Transform IiOptics Fourier Transform Ii
Optics Fourier Transform Iidiarmseven
 
Fourier series and it's examples
Fourier series and it's examplesFourier series and it's examples
Fourier series and it's examplesZorawarJaat
 
Brief Review of Fourier Analysis
Brief Review of Fourier AnalysisBrief Review of Fourier Analysis
Brief Review of Fourier Analysisop205
 
DSP_2018_FOEHU - Lec 06 - FIR Filter Design
DSP_2018_FOEHU - Lec 06 - FIR Filter DesignDSP_2018_FOEHU - Lec 06 - FIR Filter Design
DSP_2018_FOEHU - Lec 06 - FIR Filter DesignAmr E. Mohamed
 
Fourier Transform ,LAPLACE TRANSFORM,ROC and its Properties
Fourier Transform ,LAPLACE TRANSFORM,ROC and its Properties Fourier Transform ,LAPLACE TRANSFORM,ROC and its Properties
Fourier Transform ,LAPLACE TRANSFORM,ROC and its Properties Dr.SHANTHI K.G
 
DSP_FOEHU - MATLAB 01 - Discrete Time Signals and Systems
DSP_FOEHU - MATLAB 01 - Discrete Time Signals and SystemsDSP_FOEHU - MATLAB 01 - Discrete Time Signals and Systems
DSP_FOEHU - MATLAB 01 - Discrete Time Signals and SystemsAmr E. Mohamed
 
periodic functions and Fourier series
periodic functions and Fourier seriesperiodic functions and Fourier series
periodic functions and Fourier seriesUmang Gupta
 
5. fourier properties
5. fourier properties5. fourier properties
5. fourier propertiesskysunilyadav
 

What's hot (20)

DSP_FOEHU - MATLAB 02 - The Discrete-time Fourier Analysis
DSP_FOEHU - MATLAB 02 - The Discrete-time Fourier AnalysisDSP_FOEHU - MATLAB 02 - The Discrete-time Fourier Analysis
DSP_FOEHU - MATLAB 02 - The Discrete-time Fourier Analysis
 
Introduction to Fourier transform and signal analysis
Introduction to Fourier transform and signal analysisIntroduction to Fourier transform and signal analysis
Introduction to Fourier transform and signal analysis
 
Fourier transforms
Fourier transforms Fourier transforms
Fourier transforms
 
Ff tand matlab-wanjun huang
Ff tand matlab-wanjun huangFf tand matlab-wanjun huang
Ff tand matlab-wanjun huang
 
The discrete fourier transform (dsp) 4
The discrete fourier transform  (dsp) 4The discrete fourier transform  (dsp) 4
The discrete fourier transform (dsp) 4
 
An Intuitive Approach to Fourier Optics
An Intuitive Approach to Fourier OpticsAn Intuitive Approach to Fourier Optics
An Intuitive Approach to Fourier Optics
 
Data Science - Part XVI - Fourier Analysis
Data Science - Part XVI - Fourier AnalysisData Science - Part XVI - Fourier Analysis
Data Science - Part XVI - Fourier Analysis
 
DSP_FOEHU - Lec 09 - Fast Fourier Transform
DSP_FOEHU - Lec 09 - Fast Fourier TransformDSP_FOEHU - Lec 09 - Fast Fourier Transform
DSP_FOEHU - Lec 09 - Fast Fourier Transform
 
Dft and its applications
Dft and its applicationsDft and its applications
Dft and its applications
 
Dsp 2018 foehu - lec 10 - multi-rate digital signal processing
Dsp 2018 foehu - lec 10 - multi-rate digital signal processingDsp 2018 foehu - lec 10 - multi-rate digital signal processing
Dsp 2018 foehu - lec 10 - multi-rate digital signal processing
 
Optics Fourier Transform Ii
Optics Fourier Transform IiOptics Fourier Transform Ii
Optics Fourier Transform Ii
 
Fourier series and it's examples
Fourier series and it's examplesFourier series and it's examples
Fourier series and it's examples
 
Brief Review of Fourier Analysis
Brief Review of Fourier AnalysisBrief Review of Fourier Analysis
Brief Review of Fourier Analysis
 
DSP_2018_FOEHU - Lec 06 - FIR Filter Design
DSP_2018_FOEHU - Lec 06 - FIR Filter DesignDSP_2018_FOEHU - Lec 06 - FIR Filter Design
DSP_2018_FOEHU - Lec 06 - FIR Filter Design
 
Fourier Transform ,LAPLACE TRANSFORM,ROC and its Properties
Fourier Transform ,LAPLACE TRANSFORM,ROC and its Properties Fourier Transform ,LAPLACE TRANSFORM,ROC and its Properties
Fourier Transform ,LAPLACE TRANSFORM,ROC and its Properties
 
DSP_FOEHU - MATLAB 01 - Discrete Time Signals and Systems
DSP_FOEHU - MATLAB 01 - Discrete Time Signals and SystemsDSP_FOEHU - MATLAB 01 - Discrete Time Signals and Systems
DSP_FOEHU - MATLAB 01 - Discrete Time Signals and Systems
 
periodic functions and Fourier series
periodic functions and Fourier seriesperiodic functions and Fourier series
periodic functions and Fourier series
 
DSP 08 _ Sheet Eight
DSP 08 _ Sheet EightDSP 08 _ Sheet Eight
DSP 08 _ Sheet Eight
 
5. fourier properties
5. fourier properties5. fourier properties
5. fourier properties
 
Fourier Transform
Fourier TransformFourier Transform
Fourier Transform
 

Viewers also liked

GEOGRAPHIC INFORMATION SYSTEM (GIS) & REMOTE SENSING (RS)
GEOGRAPHIC  INFORMATION  SYSTEM (GIS)  & REMOTE SENSING  (RS)GEOGRAPHIC  INFORMATION  SYSTEM (GIS)  & REMOTE SENSING  (RS)
GEOGRAPHIC INFORMATION SYSTEM (GIS) & REMOTE SENSING (RS)Harish kumar Lekkala
 
Unit 7 envt_policy_legislation_rules_and_regulations
Unit 7 envt_policy_legislation_rules_and_regulationsUnit 7 envt_policy_legislation_rules_and_regulations
Unit 7 envt_policy_legislation_rules_and_regulationsHarish kumar Lekkala
 
RFID based Highway Toll Fee Process System
RFID based Highway Toll Fee Process SystemRFID based Highway Toll Fee Process System
RFID based Highway Toll Fee Process SystemAmar Reddy
 
Indian environmental legislation
Indian environmental legislationIndian environmental legislation
Indian environmental legislationmona verma
 
Rural Roads as Infrastructure for Economic Development
Rural Roads as Infrastructure for Economic DevelopmentRural Roads as Infrastructure for Economic Development
Rural Roads as Infrastructure for Economic DevelopmentIram Aziz
 
Influence of Micro Silica and GGBS on mechanical properties on high strength...
Influence of  Micro Silica and GGBS on mechanical properties on high strength...Influence of  Micro Silica and GGBS on mechanical properties on high strength...
Influence of Micro Silica and GGBS on mechanical properties on high strength...Harish kumar Lekkala
 
Agricultural drone
Agricultural droneAgricultural drone
Agricultural droneashishya30
 
Environmental legislations in India
Environmental legislations in IndiaEnvironmental legislations in India
Environmental legislations in IndiaShubhranshu Upadhyay
 
Geographic information system(gis)
Geographic information system(gis)Geographic information system(gis)
Geographic information system(gis)MapQuest.Inc
 
Unit 6 environmental_impact_assessment
Unit 6 environmental_impact_assessmentUnit 6 environmental_impact_assessment
Unit 6 environmental_impact_assessmentHarish kumar Lekkala
 
Unit 5 global_environmental_problems_and_global_efforts
Unit 5 global_environmental_problems_and_global_effortsUnit 5 global_environmental_problems_and_global_efforts
Unit 5 global_environmental_problems_and_global_effortsHarish kumar Lekkala
 
Gis (geographic information system)
Gis (geographic information system)Gis (geographic information system)
Gis (geographic information system)Saad Bare
 

Viewers also liked (20)

GEOGRAPHIC INFORMATION SYSTEM (GIS) & REMOTE SENSING (RS)
GEOGRAPHIC  INFORMATION  SYSTEM (GIS)  & REMOTE SENSING  (RS)GEOGRAPHIC  INFORMATION  SYSTEM (GIS)  & REMOTE SENSING  (RS)
GEOGRAPHIC INFORMATION SYSTEM (GIS) & REMOTE SENSING (RS)
 
Unit 8 towards_sustainable_future
Unit 8 towards_sustainable_futureUnit 8 towards_sustainable_future
Unit 8 towards_sustainable_future
 
Unit 7 envt_policy_legislation_rules_and_regulations
Unit 7 envt_policy_legislation_rules_and_regulationsUnit 7 envt_policy_legislation_rules_and_regulations
Unit 7 envt_policy_legislation_rules_and_regulations
 
RFID based Highway Toll Fee Process System
RFID based Highway Toll Fee Process SystemRFID based Highway Toll Fee Process System
RFID based Highway Toll Fee Process System
 
Unit 1 ecosystem
Unit 1 ecosystemUnit 1 ecosystem
Unit 1 ecosystem
 
AN M.ENG PRESENTATION ON ROAD NETWORK
AN M.ENG PRESENTATION ON ROAD NETWORKAN M.ENG PRESENTATION ON ROAD NETWORK
AN M.ENG PRESENTATION ON ROAD NETWORK
 
Indian environmental legislation
Indian environmental legislationIndian environmental legislation
Indian environmental legislation
 
Joy
JoyJoy
Joy
 
Rural Roads as Infrastructure for Economic Development
Rural Roads as Infrastructure for Economic DevelopmentRural Roads as Infrastructure for Economic Development
Rural Roads as Infrastructure for Economic Development
 
Influence of Micro Silica and GGBS on mechanical properties on high strength...
Influence of  Micro Silica and GGBS on mechanical properties on high strength...Influence of  Micro Silica and GGBS on mechanical properties on high strength...
Influence of Micro Silica and GGBS on mechanical properties on high strength...
 
Agricultural drone
Agricultural droneAgricultural drone
Agricultural drone
 
Environmental legislations in India
Environmental legislations in IndiaEnvironmental legislations in India
Environmental legislations in India
 
Desalination of sea water
Desalination of sea waterDesalination of sea water
Desalination of sea water
 
Geographic information system(gis)
Geographic information system(gis)Geographic information system(gis)
Geographic information system(gis)
 
Unit 6 environmental_impact_assessment
Unit 6 environmental_impact_assessmentUnit 6 environmental_impact_assessment
Unit 6 environmental_impact_assessment
 
Unit 3 biodiversity
Unit 3 biodiversityUnit 3 biodiversity
Unit 3 biodiversity
 
Unit 2 nat_resources
Unit 2 nat_resourcesUnit 2 nat_resources
Unit 2 nat_resources
 
Unit 5 global_environmental_problems_and_global_efforts
Unit 5 global_environmental_problems_and_global_effortsUnit 5 global_environmental_problems_and_global_efforts
Unit 5 global_environmental_problems_and_global_efforts
 
Gis (geographic information system)
Gis (geographic information system)Gis (geographic information system)
Gis (geographic information system)
 
Global warming
Global warmingGlobal warming
Global warming
 

Similar to Unit 8

UNIT-2 DSP.ppt
UNIT-2 DSP.pptUNIT-2 DSP.ppt
UNIT-2 DSP.pptshailaja68
 
Fourier-Series_FT_Laplace-Transform_Letures_Regular_F-for-Students_10-1-1.ppt
Fourier-Series_FT_Laplace-Transform_Letures_Regular_F-for-Students_10-1-1.pptFourier-Series_FT_Laplace-Transform_Letures_Regular_F-for-Students_10-1-1.ppt
Fourier-Series_FT_Laplace-Transform_Letures_Regular_F-for-Students_10-1-1.pptMozammelHossain31
 
Ch7 noise variation of different modulation scheme pg 63
Ch7 noise variation of different modulation scheme pg 63Ch7 noise variation of different modulation scheme pg 63
Ch7 noise variation of different modulation scheme pg 63Prateek Omer
 
Ch1 representation of signal pg 130
Ch1 representation of signal pg 130Ch1 representation of signal pg 130
Ch1 representation of signal pg 130Prateek Omer
 
Noise performence
Noise performenceNoise performence
Noise performencePunk Pankaj
 
Fourier supplementals
Fourier supplementalsFourier supplementals
Fourier supplementalsPartha_bappa
 
signal and system Lecture 2
signal and system Lecture 2signal and system Lecture 2
signal and system Lecture 2iqbal ahmad
 
DIGITAL SIGNAL PROCESSING: Sampling and Reconstruction on MATLAB
DIGITAL SIGNAL PROCESSING: Sampling and Reconstruction on MATLABDIGITAL SIGNAL PROCESSING: Sampling and Reconstruction on MATLAB
DIGITAL SIGNAL PROCESSING: Sampling and Reconstruction on MATLABMartin Wachiye Wafula
 
A Model of Partial Tracks for Tension-Modulated Steel-String Guitar Tones
A Model of Partial Tracks for Tension-Modulated Steel-String Guitar TonesA Model of Partial Tracks for Tension-Modulated Steel-String Guitar Tones
A Model of Partial Tracks for Tension-Modulated Steel-String Guitar TonesMatthieu Hodgkinson
 
Accurate Evaluation of Interharmonics of a Six Pulse, Full Wave - Three Phase...
Accurate Evaluation of Interharmonics of a Six Pulse, Full Wave - Three Phase...Accurate Evaluation of Interharmonics of a Six Pulse, Full Wave - Three Phase...
Accurate Evaluation of Interharmonics of a Six Pulse, Full Wave - Three Phase...idescitation
 
Speech signal time frequency representation
Speech signal time frequency representationSpeech signal time frequency representation
Speech signal time frequency representationNikolay Karpov
 
On Fractional Fourier Transform Moments Based On Ambiguity Function
On Fractional Fourier Transform Moments Based On Ambiguity FunctionOn Fractional Fourier Transform Moments Based On Ambiguity Function
On Fractional Fourier Transform Moments Based On Ambiguity FunctionCSCJournals
 
DSP_2018_FOEHU - Lec 02 - Sampling of Continuous Time Signals
DSP_2018_FOEHU - Lec 02 - Sampling of Continuous Time SignalsDSP_2018_FOEHU - Lec 02 - Sampling of Continuous Time Signals
DSP_2018_FOEHU - Lec 02 - Sampling of Continuous Time SignalsAmr E. Mohamed
 
Eece 301 note set 14 fourier transform
Eece 301 note set 14 fourier transformEece 301 note set 14 fourier transform
Eece 301 note set 14 fourier transformSandilya Sridhara
 

Similar to Unit 8 (20)

UNIT-2 DSP.ppt
UNIT-2 DSP.pptUNIT-2 DSP.ppt
UNIT-2 DSP.ppt
 
RES701 Research Methodology_FFT
RES701 Research Methodology_FFTRES701 Research Methodology_FFT
RES701 Research Methodology_FFT
 
Fourier-Series_FT_Laplace-Transform_Letures_Regular_F-for-Students_10-1-1.ppt
Fourier-Series_FT_Laplace-Transform_Letures_Regular_F-for-Students_10-1-1.pptFourier-Series_FT_Laplace-Transform_Letures_Regular_F-for-Students_10-1-1.ppt
Fourier-Series_FT_Laplace-Transform_Letures_Regular_F-for-Students_10-1-1.ppt
 
Ch7 noise variation of different modulation scheme pg 63
Ch7 noise variation of different modulation scheme pg 63Ch7 noise variation of different modulation scheme pg 63
Ch7 noise variation of different modulation scheme pg 63
 
unit 4,5 (1).docx
unit 4,5 (1).docxunit 4,5 (1).docx
unit 4,5 (1).docx
 
Ch1 representation of signal pg 130
Ch1 representation of signal pg 130Ch1 representation of signal pg 130
Ch1 representation of signal pg 130
 
noise
noisenoise
noise
 
Noise performence
Noise performenceNoise performence
Noise performence
 
Fourier supplementals
Fourier supplementalsFourier supplementals
Fourier supplementals
 
signal and system Lecture 2
signal and system Lecture 2signal and system Lecture 2
signal and system Lecture 2
 
DIGITAL SIGNAL PROCESSING: Sampling and Reconstruction on MATLAB
DIGITAL SIGNAL PROCESSING: Sampling and Reconstruction on MATLABDIGITAL SIGNAL PROCESSING: Sampling and Reconstruction on MATLAB
DIGITAL SIGNAL PROCESSING: Sampling and Reconstruction on MATLAB
 
A Model of Partial Tracks for Tension-Modulated Steel-String Guitar Tones
A Model of Partial Tracks for Tension-Modulated Steel-String Guitar TonesA Model of Partial Tracks for Tension-Modulated Steel-String Guitar Tones
A Model of Partial Tracks for Tension-Modulated Steel-String Guitar Tones
 
Kanal wireless dan propagasi
Kanal wireless dan propagasiKanal wireless dan propagasi
Kanal wireless dan propagasi
 
Accurate Evaluation of Interharmonics of a Six Pulse, Full Wave - Three Phase...
Accurate Evaluation of Interharmonics of a Six Pulse, Full Wave - Three Phase...Accurate Evaluation of Interharmonics of a Six Pulse, Full Wave - Three Phase...
Accurate Evaluation of Interharmonics of a Six Pulse, Full Wave - Three Phase...
 
Speech signal time frequency representation
Speech signal time frequency representationSpeech signal time frequency representation
Speech signal time frequency representation
 
M4.pdf
M4.pdfM4.pdf
M4.pdf
 
On Fractional Fourier Transform Moments Based On Ambiguity Function
On Fractional Fourier Transform Moments Based On Ambiguity FunctionOn Fractional Fourier Transform Moments Based On Ambiguity Function
On Fractional Fourier Transform Moments Based On Ambiguity Function
 
Nt lecture skm-iiit-bh
Nt lecture skm-iiit-bhNt lecture skm-iiit-bh
Nt lecture skm-iiit-bh
 
DSP_2018_FOEHU - Lec 02 - Sampling of Continuous Time Signals
DSP_2018_FOEHU - Lec 02 - Sampling of Continuous Time SignalsDSP_2018_FOEHU - Lec 02 - Sampling of Continuous Time Signals
DSP_2018_FOEHU - Lec 02 - Sampling of Continuous Time Signals
 
Eece 301 note set 14 fourier transform
Eece 301 note set 14 fourier transformEece 301 note set 14 fourier transform
Eece 301 note set 14 fourier transform
 

Recently uploaded

Tamil Call Girls Bhayandar WhatsApp +91-9930687706, Best Service
Tamil Call Girls Bhayandar WhatsApp +91-9930687706, Best ServiceTamil Call Girls Bhayandar WhatsApp +91-9930687706, Best Service
Tamil Call Girls Bhayandar WhatsApp +91-9930687706, Best Servicemeghakumariji156
 
Introduction to Data Visualization,Matplotlib.pdf
Introduction to Data Visualization,Matplotlib.pdfIntroduction to Data Visualization,Matplotlib.pdf
Introduction to Data Visualization,Matplotlib.pdfsumitt6_25730773
 
457503602-5-Gas-Well-Testing-and-Analysis-pptx.pptx
457503602-5-Gas-Well-Testing-and-Analysis-pptx.pptx457503602-5-Gas-Well-Testing-and-Analysis-pptx.pptx
457503602-5-Gas-Well-Testing-and-Analysis-pptx.pptxrouholahahmadi9876
 
Ground Improvement Technique: Earth Reinforcement
Ground Improvement Technique: Earth ReinforcementGround Improvement Technique: Earth Reinforcement
Ground Improvement Technique: Earth ReinforcementDr. Deepak Mudgal
 
Double Revolving field theory-how the rotor develops torque
Double Revolving field theory-how the rotor develops torqueDouble Revolving field theory-how the rotor develops torque
Double Revolving field theory-how the rotor develops torqueBhangaleSonal
 
Basic Electronics for diploma students as per technical education Kerala Syll...
Basic Electronics for diploma students as per technical education Kerala Syll...Basic Electronics for diploma students as per technical education Kerala Syll...
Basic Electronics for diploma students as per technical education Kerala Syll...ppkakm
 
Hospital management system project report.pdf
Hospital management system project report.pdfHospital management system project report.pdf
Hospital management system project report.pdfKamal Acharya
 
fitting shop and tools used in fitting shop .ppt
fitting shop and tools used in fitting shop .pptfitting shop and tools used in fitting shop .ppt
fitting shop and tools used in fitting shop .pptAfnanAhmad53
 
Max. shear stress theory-Maximum Shear Stress Theory ​ Maximum Distortional ...
Max. shear stress theory-Maximum Shear Stress Theory ​  Maximum Distortional ...Max. shear stress theory-Maximum Shear Stress Theory ​  Maximum Distortional ...
Max. shear stress theory-Maximum Shear Stress Theory ​ Maximum Distortional ...ronahami
 
Theory of Time 2024 (Universal Theory for Everything)
Theory of Time 2024 (Universal Theory for Everything)Theory of Time 2024 (Universal Theory for Everything)
Theory of Time 2024 (Universal Theory for Everything)Ramkumar k
 
Thermal Engineering -unit - III & IV.ppt
Thermal Engineering -unit - III & IV.pptThermal Engineering -unit - III & IV.ppt
Thermal Engineering -unit - III & IV.pptDineshKumar4165
 
Standard vs Custom Battery Packs - Decoding the Power Play
Standard vs Custom Battery Packs - Decoding the Power PlayStandard vs Custom Battery Packs - Decoding the Power Play
Standard vs Custom Battery Packs - Decoding the Power PlayEpec Engineered Technologies
 
Introduction to Serverless with AWS Lambda
Introduction to Serverless with AWS LambdaIntroduction to Serverless with AWS Lambda
Introduction to Serverless with AWS LambdaOmar Fathy
 
XXXXXXXXXXXXXXXXXXXXXXXXXXXXXXXXXXXXXXXXXXXXXXXXXXXX
XXXXXXXXXXXXXXXXXXXXXXXXXXXXXXXXXXXXXXXXXXXXXXXXXXXXXXXXXXXXXXXXXXXXXXXXXXXXXXXXXXXXXXXXXXXXXXXXXXXXXXXX
XXXXXXXXXXXXXXXXXXXXXXXXXXXXXXXXXXXXXXXXXXXXXXXXXXXXssuser89054b
 
Orlando’s Arnold Palmer Hospital Layout Strategy-1.pptx
Orlando’s Arnold Palmer Hospital Layout Strategy-1.pptxOrlando’s Arnold Palmer Hospital Layout Strategy-1.pptx
Orlando’s Arnold Palmer Hospital Layout Strategy-1.pptxMuhammadAsimMuhammad6
 
Design For Accessibility: Getting it right from the start
Design For Accessibility: Getting it right from the startDesign For Accessibility: Getting it right from the start
Design For Accessibility: Getting it right from the startQuintin Balsdon
 
1_Introduction + EAM Vocabulary + how to navigate in EAM.pdf
1_Introduction + EAM Vocabulary + how to navigate in EAM.pdf1_Introduction + EAM Vocabulary + how to navigate in EAM.pdf
1_Introduction + EAM Vocabulary + how to navigate in EAM.pdfAldoGarca30
 
HOA1&2 - Module 3 - PREHISTORCI ARCHITECTURE OF KERALA.pptx
HOA1&2 - Module 3 - PREHISTORCI ARCHITECTURE OF KERALA.pptxHOA1&2 - Module 3 - PREHISTORCI ARCHITECTURE OF KERALA.pptx
HOA1&2 - Module 3 - PREHISTORCI ARCHITECTURE OF KERALA.pptxSCMS School of Architecture
 
UNIT 4 PTRP final Convergence in probability.pptx
UNIT 4 PTRP final Convergence in probability.pptxUNIT 4 PTRP final Convergence in probability.pptx
UNIT 4 PTRP final Convergence in probability.pptxkalpana413121
 

Recently uploaded (20)

Tamil Call Girls Bhayandar WhatsApp +91-9930687706, Best Service
Tamil Call Girls Bhayandar WhatsApp +91-9930687706, Best ServiceTamil Call Girls Bhayandar WhatsApp +91-9930687706, Best Service
Tamil Call Girls Bhayandar WhatsApp +91-9930687706, Best Service
 
Introduction to Data Visualization,Matplotlib.pdf
Introduction to Data Visualization,Matplotlib.pdfIntroduction to Data Visualization,Matplotlib.pdf
Introduction to Data Visualization,Matplotlib.pdf
 
457503602-5-Gas-Well-Testing-and-Analysis-pptx.pptx
457503602-5-Gas-Well-Testing-and-Analysis-pptx.pptx457503602-5-Gas-Well-Testing-and-Analysis-pptx.pptx
457503602-5-Gas-Well-Testing-and-Analysis-pptx.pptx
 
Ground Improvement Technique: Earth Reinforcement
Ground Improvement Technique: Earth ReinforcementGround Improvement Technique: Earth Reinforcement
Ground Improvement Technique: Earth Reinforcement
 
FEA Based Level 3 Assessment of Deformed Tanks with Fluid Induced Loads
FEA Based Level 3 Assessment of Deformed Tanks with Fluid Induced LoadsFEA Based Level 3 Assessment of Deformed Tanks with Fluid Induced Loads
FEA Based Level 3 Assessment of Deformed Tanks with Fluid Induced Loads
 
Double Revolving field theory-how the rotor develops torque
Double Revolving field theory-how the rotor develops torqueDouble Revolving field theory-how the rotor develops torque
Double Revolving field theory-how the rotor develops torque
 
Basic Electronics for diploma students as per technical education Kerala Syll...
Basic Electronics for diploma students as per technical education Kerala Syll...Basic Electronics for diploma students as per technical education Kerala Syll...
Basic Electronics for diploma students as per technical education Kerala Syll...
 
Hospital management system project report.pdf
Hospital management system project report.pdfHospital management system project report.pdf
Hospital management system project report.pdf
 
fitting shop and tools used in fitting shop .ppt
fitting shop and tools used in fitting shop .pptfitting shop and tools used in fitting shop .ppt
fitting shop and tools used in fitting shop .ppt
 
Max. shear stress theory-Maximum Shear Stress Theory ​ Maximum Distortional ...
Max. shear stress theory-Maximum Shear Stress Theory ​  Maximum Distortional ...Max. shear stress theory-Maximum Shear Stress Theory ​  Maximum Distortional ...
Max. shear stress theory-Maximum Shear Stress Theory ​ Maximum Distortional ...
 
Theory of Time 2024 (Universal Theory for Everything)
Theory of Time 2024 (Universal Theory for Everything)Theory of Time 2024 (Universal Theory for Everything)
Theory of Time 2024 (Universal Theory for Everything)
 
Thermal Engineering -unit - III & IV.ppt
Thermal Engineering -unit - III & IV.pptThermal Engineering -unit - III & IV.ppt
Thermal Engineering -unit - III & IV.ppt
 
Standard vs Custom Battery Packs - Decoding the Power Play
Standard vs Custom Battery Packs - Decoding the Power PlayStandard vs Custom Battery Packs - Decoding the Power Play
Standard vs Custom Battery Packs - Decoding the Power Play
 
Introduction to Serverless with AWS Lambda
Introduction to Serverless with AWS LambdaIntroduction to Serverless with AWS Lambda
Introduction to Serverless with AWS Lambda
 
XXXXXXXXXXXXXXXXXXXXXXXXXXXXXXXXXXXXXXXXXXXXXXXXXXXX
XXXXXXXXXXXXXXXXXXXXXXXXXXXXXXXXXXXXXXXXXXXXXXXXXXXXXXXXXXXXXXXXXXXXXXXXXXXXXXXXXXXXXXXXXXXXXXXXXXXXXXXX
XXXXXXXXXXXXXXXXXXXXXXXXXXXXXXXXXXXXXXXXXXXXXXXXXXXX
 
Orlando’s Arnold Palmer Hospital Layout Strategy-1.pptx
Orlando’s Arnold Palmer Hospital Layout Strategy-1.pptxOrlando’s Arnold Palmer Hospital Layout Strategy-1.pptx
Orlando’s Arnold Palmer Hospital Layout Strategy-1.pptx
 
Design For Accessibility: Getting it right from the start
Design For Accessibility: Getting it right from the startDesign For Accessibility: Getting it right from the start
Design For Accessibility: Getting it right from the start
 
1_Introduction + EAM Vocabulary + how to navigate in EAM.pdf
1_Introduction + EAM Vocabulary + how to navigate in EAM.pdf1_Introduction + EAM Vocabulary + how to navigate in EAM.pdf
1_Introduction + EAM Vocabulary + how to navigate in EAM.pdf
 
HOA1&2 - Module 3 - PREHISTORCI ARCHITECTURE OF KERALA.pptx
HOA1&2 - Module 3 - PREHISTORCI ARCHITECTURE OF KERALA.pptxHOA1&2 - Module 3 - PREHISTORCI ARCHITECTURE OF KERALA.pptx
HOA1&2 - Module 3 - PREHISTORCI ARCHITECTURE OF KERALA.pptx
 
UNIT 4 PTRP final Convergence in probability.pptx
UNIT 4 PTRP final Convergence in probability.pptxUNIT 4 PTRP final Convergence in probability.pptx
UNIT 4 PTRP final Convergence in probability.pptx
 

Unit 8

  • 1. S&S-8 (The z-transform) 1 of 49 Dr. Ravi Billa Signals & Systems – 8 August 5, 2011 VIII. The z-transform Syllabus: Fundamental difference between continuous and discrete time signals, Discrete time signal representation using complex exponential and sinusoidal components, Periodicity of discrete time signal using complex exponential signal, Concept of z-transform of a discrete sequence, Distinction between Laplace, Fourier and z transforms, Region of convergence in z-transform, Constraints on ROC for various classes of signals, Inverse z-transform, Properties of z-transforms. O&W Ch. 10. Contents: 8.1 Continuous-time signals 8.2 Discrete-time signals 8.3 The z-transform 8.4 Transforms of some useful sequences 8.5 Important properties of z-transforms 8.6 Transforms of some useful sequences, cont’d. 8.7 Region of convergence (ROC) 8.8 Inverse z-transform by partial fractions 8.9 Inverse z-transform by power series expansion (long division) www.jntuworld.com www.jntuworld.com
  • 2. S&S-8 (The z-transform) 2 of 49 Dr. Ravi Billa 8.1 Continuous-time signals Continuous-time signals are functions of the continuous time variable, t. In this course the independent variable is time; however, this need not always be the case. Example 1 [Sinusoidal signal] The household voltage in India is 230 Volts (rms), 50 Hertz or cycles per second. The waveform may be expressed in various forms, for example, ( ) √ , or ( ) √ The period is 1/50 second. The amplitude is √ volts, the radian frequency is rad/sec., the Hertz (or cyclic) frequency is = 50 Hz. Quiz What are the amplitude, rms value, period and frequency of ( ) ? Periodic and aperiodic signals A signal ( )is periodic if and only if ( ) ( ) where is the period. The smallest value such that the above equation is satisfied is called the fundamental period or simply the period. Any deterministic signal not satisfying the above equation is called aperiodic. If a signal is periodic with a fundamental period of , then it is also periodic with a period of , … or any integer multiple of . Note We use the symbols (Hz) and (radians/second) for analog frequencies. Example 2 [Periodicity] For the signal ( ) ( ) ( ) the parameter A is the amplitude (peak value), F0 is the frequency (Hertz or cycles per second), and θ or ( ) is the phase. The radian frequency is rad/sec. The period is given by ⁄ .The periodicity of the above ( )– whether it is periodic or not – can be verified by checking if ( ( )) ( ) Now, replacing by , we have ( ( )) ( ( ) ) ( ) [ ( ) ( ) ] ( ) ( ) Note that is the smallest value of such that ( ) ( ). There are no restrictions on the period ( ) or the frequency ( ) in this analog case (as there are in the case of the discrete- time counterpart). The sum of two or more continuous-time sinusoids may or may not be periodic, depending on the relationships among their respective periods (or frequencies). If the ratio of their periods (or frequencies) can be expressed as a rational number, their sum will be a periodic signal. For instance, the sum of two sinusoids of frequencies and is periodic if ⁄ = ⁄ where the ’s are integers. www.jntuworld.com www.jntuworld.com
  • 3. S&S-8 (The z-transform) 3 of 49 Dr. Ravi Billa Another way of putting this is: if their frequencies are commensurable, their sum will be a periodic signal. Two frequencies, F1 and F2, are commensurable if they have a common measure. That is, there is a number contained in each an integral number of times. Thus, if is the smallest such number, and is called the fundamental frequency. The periods, and corresponding to and are therefore related by which is rational. HW Determine if the signal ( ) ( ) ( ) is periodic and if so determine its frequency. [See BasicSim-51] Note that if the signals ( ) and ( ) have the same period , then ( ) ( ) ( ) also is periodic with the same period . That is, linear operations (addition in this case) do not affect the periodicity of the resulting signal. In contrast nonlinear operations, generally, can produce different frequencies (or periodicities). See example below. Example 3 [Product of two sinusoids] Multiplication is a nonlinear operation. If ( ) and ( ) , then the signal ( ) ( ) ( ) will contain the sum and difference frequencies, ( ) and ( ), and the original frequencies and are absent. A similar observation holds for the signal ( ) = ( ) or ( ). However, if ( ) and ( ) have different (commensurable) frequencies then a linear combination may produce new frequency(s): an example is ( ) ( ) ( ). HW Recast the linear sum signal ( ) ( ) ( ) as the product of two sinusoids. [See BasicSim-51] Real exponential signal Consider ( ) where and are constants. For example, ( ) or ( ) . Sketch the waveforms. How do you find the time constant? Put the exponential in the form of . For example, ( ) . Complex exponential signal In what follows we make extensive use of Euler’s theorem and the corresponding formulae ( )⁄ ( )⁄ It is often mathematically convenient to represent real signals in terms of complex quantities. Given the phasor (or complex number) | | | | , the real sinusoidal signal ( ) can be expressed in terms of the complex signal ̃( ) . In particular ( ) is defined as the real part of ̃( )as ( ) {̃( )} { } ( ) {| | } www.jntuworld.com www.jntuworld.com
  • 4. S&S-8 (The z-transform) 4 of 49 Dr. Ravi Billa {| | ( ) } | | ( ) The complex signal ̃( ) | | ( ) is referred to as a rotating phasor characterized by the three parameters: (1) The amplitude, | |, (2) The phase, , and (3) The radian frequency . The signal ̃( ) is periodic with period . Alternatively, we may relate ̃( ) to its sinusoidal counterpart by writing ( ) ̃( ) ̃ ( ) | | ( ) | | ( ) ( ) which is a representation in terms of conjugate, oppositely rotating phasors. The expressions (1) and (2) for ( ) | | ( ) are referred to as time-domain representations. An alternative representation for x(t) is provided in the frequency domain. Since ̃( ) | | ( ) is completely specified by | | and for a given value of (or ), this alternative frequency domain representation can take the form of two plots, one showing the amplitude | | as a function of frequency, , and the other showing the phase as a function of . The result is the single-sided spectrum shown here. Another version of the frequency domain representation, the two-sided spectrum, results if we make a spectral plot corresponding to Eq. (2). The two-sided spectrum has lines at both and as shown below. Note the even symmetry of the amplitude plot and the odd symmetry of the phase plot. F F0 Phase  – –F0 |A|/2 F –F0 Amplitude |A|/2 F0  F0 Phase F0 |A| F 0 Amplitude F0 www.jntuworld.com www.jntuworld.com
  • 5. S&S-8 (The z-transform) 5 of 49 Dr. Ravi Billa 8.2 Discrete-time signals Discrete-time signals are functions of the discrete time variable, n. In this course the independent variable is time; however, this need not always be the case. If a continuous-time signal x(t) is sampled at T-second intervals, the result is a discrete-time signal or sequence { ( )} where n is an index on the sampling instants. For convenience we shall drop the T and the braces and use just x(n) to represent the sequence. Example 1 Consider the continuous-time waveform with a frequency of 4 Hz given by ( ) Sampling the above signal at T-second intervals results in the discrete-time signal ( ). Mathematically, this is accomplished by replacing by and the expression for ( ) is given by ( ) If, for instance, the sampling frequency is 16 Hz then T = 1/16 and the discrete-time signal becomes ( ) ( ⁄ ) ( ) In the notation ( ) we may drop the symbol T and refer to the discrete-time signal simply as ( ) ( ). Quiz Write expressions for ( ) and ( ) obtained by sampling, respectively, ( ) and ( ) at 16 Hz. Sketch and label the sequences ( ), ( ) and ( ) for n = 0 to 12. www.jntuworld.com www.jntuworld.com
  • 6. S&S-8 (The z-transform) 6 of 49 Dr. Ravi Billa MATLAB plotting of continuous-time function %Plot x1(t) = cos 2π4t as t goes from 0 to 0.5 sec (2 cycles) in steps of T = 1/160 sec. t = 0: 1/160: 0.5; x1 = cos (2*pi*4*t); plot(t,x1); xlabel ('Time, t, seconds'), ylabel('x1(t)'); title ('x1(t) = cos (2*pi*4*t) – 4Hz continuous Cosine plotted at T = 1/160 sec.') HW Explain the effect of changing “plot(t, x1)” in the above program segment to “plot(t, x1, 'bo')”. %Plot x1(t) = cos 2π4t as t goes from 0 to 0.5 sec (2 cycles) in steps of T = 1/160 sec. t = 0: 1/160: 0.5; x1 = cos (2*pi*4*t); plot(t,x1, 'bo'); xlabel ('Time, t, seconds'), ylabel('x1(t)'); title ('x1(t) = cos (2*pi*4*t) – 4Hz continuous Cosine plotted at T = 1/160 sec.') 0 0.05 0.1 0.15 0.2 0.25 0.3 0.35 0.4 0.45 0.5 -1 -0.8 -0.6 -0.4 -0.2 0 0.2 0.4 0.6 0.8 1 Time, t, seconds x1(t) x1(t) = cos (2*pi*4*t) – 4Hz continuous Cosine plotted at T = 1/160 sec. www.jntuworld.com www.jntuworld.com
  • 7. S&S-8 (The z-transform) 7 of 49 Dr. Ravi Billa MATLAB plotting of discrete-time function %Plot x1(n) = cos nπ/20 as n goes from 0 to 80 (2 cycles) in steps of 1 (T = 1/160 sec.) n = 0: 1: 80; x1 = cos (n*pi/20); plot(n, x1, 'bo');grid %'bo' = Blue circles xlabel ('Sample number, n'), ylabel('x1(n)'); title ('x1(n) = cos (n*pi/20) – 4Hz sampled Cosine at 160 samples/sec.') HW Explain the result of changing “plot(n, x1, 'bo')” in the above program segment to “plot(n, x1)”. %Plot x1(n) = cos nπ/20 as n goes from 0 to 80 (2 cycles) in steps of 1 (T = 1/160 sec.) n = 0: 1: 80; x1 = cos (n*pi/20); plot(n, x1);grid xlabel ('Sample number, n'), ylabel('x1(n)'); title ('x1(n) = cos (n*pi/20) – 4Hz sampled Cosine at 160 samples/sec.') 0 10 20 30 40 50 60 70 80 -1 -0.8 -0.6 -0.4 -0.2 0 0.2 0.4 0.6 0.8 1 Sample number, n x1(n) x1(n) = cos (n*pi/20) – 4Hz sampled Cosine at 160 samples/sec. www.jntuworld.com www.jntuworld.com
  • 8. S&S-8 (The z-transform) 8 of 49 Dr. Ravi Billa MATLAB plotting of discrete-time function – the stem plot At each sampling instant, n, a vertical line (stem) is erected with a height equal to the sample value. %Stem plot x1(n) = cos nπ/20 as n goes from 0 to 80 (2 cycles) in steps of 1 %(T = 1/160 sec.) n = 0: 1: 80; x1 = cos (n*pi/20); stem(n,x1) xlabel ('Sample number, n'), ylabel('x1(n)'); title ('x1(n) = cos (n*pi/20) – Stem plot of 4Hz Cosine sampled at 160 Hz.') 0 10 20 30 40 50 60 70 80 -1 -0.8 -0.6 -0.4 -0.2 0 0.2 0.4 0.6 0.8 1 Sample number, n x1(n) x1(n) = cos (n*pi/20) – Stem plot of 4Hz Cosine sampled at 160 Hz. www.jntuworld.com www.jntuworld.com
  • 9. S&S-8 (The z-transform) 9 of 49 Dr. Ravi Billa %Difference between plotting with 'bo' versus 'b' n = 0: 1: 80; % %'bo' = Data points are marked with blue circles x1 = cos (n*pi/20); subplot(2, 1, 1), plot(n, x1, 'bo'); %subplot(2, 1, 1) – 2 rows, 1 column, #1 xlabel ('n'), ylabel('x1(n)'); title ('Sub-plot(2,1,1): 1st Row, 1st Column, Plot #1'); legend ('Legend 1'); grid; % %'b' = Data points are shown as a blue connected (continuous) line!! subplot(2, 1, 2), plot(n, cos (n*pi/20), 'b'); %subplot(2, 1, 2) – 2 rows, 1 column, #2 xlabel ('n'), ylabel('x2(n)'); title ('Sub-plot(2,1,2): 2nd Row, 1st Column, Plot #2'); legend ('Legend 2'); %No grid 0 10 20 30 40 50 60 70 80 -1 -0.5 0 0.5 1 n x1(n) Sub-plot(2,1,1): 1st Row, 1st Column, Plot #1 Legend 1 0 10 20 30 40 50 60 70 80 -1 -0.5 0 0.5 1 n x2(n) Sub-plot(2,1,2): 2nd Row, 1st Column, Plot #2 Legend 2 www.jntuworld.com www.jntuworld.com
  • 10. S&S-8 (The z-transform) 10 of 49 Dr. Ravi Billa Example 3 [The real exponential sequence] The continuous-time real-valued exponential function, when sampled, results in the real-valued exponential sequence also known as a geometric series. Consider the familiar continuous time signal defined for positive time ( ) The sampled version is given by setting t = nT ( ) ( ) Dropping the T from ( ) and setting we can write the positive time sequence as ( ) Using the unit step sequence ( ), the sequence ( ) may also be expressed as ( ) ( ) (The sequence can also be defined for both positive and negative n, by simply writing ( ) for all n.) 0 1 2 3 4 5 6 x(n) = an u(n) n 1 a2 a3 a4 a a = T e  0 1 2 3 4 5 6 x(t) = t e  , t  0 t 1 www.jntuworld.com www.jntuworld.com
  • 11. S&S-8 (The z-transform) 11 of 49 Dr. Ravi Billa MATLAB plots of ( ) = ( ) and ( )= ( ) K=2; alpha = 5; T=0.05; %Continuous-time. Plot x(t) = K*exp(-alpha*t) for t = 0 to 1 sec. t = 0: 1/100: 1;xt = K*exp(-alpha*t) ; subplot(2,1,1);plot(t, xt, 'k');xlabel ('t'), ylabel('xt'); legend ('xt = K*exp(-alpha*t) for t = 0 to 1 sec.'); title ('Continuous-time'); grid; % %Discrete-time.Plotx(n) = K*exp(-alpha*n*T) for n = 0 to 20. n = 0: 20; xn = K*exp(-alpha*n*T) ; subplot(2,1,2);stem(n, xn, 'b'); xlabel ('n'), ylabel('xn'); legend ('xn = K*exp(-alpha*n*T) for n = 0 to 20'); title ('Discrete-time'); grid; 0 0.1 0.2 0.3 0.4 0.5 0.6 0.7 0.8 0.9 1 0 0.5 1 1.5 2 t xt Continuous-time xt = K*exp(-alpha*t) for t = 0 to 1 sec. 0 2 4 6 8 10 12 14 16 18 20 0 0.5 1 1.5 2 n xn Discrete-time xn = K*exp(-alpha*n*T) for n = 0 to 20 www.jntuworld.com www.jntuworld.com
  • 12. S&S-8 (The z-transform) 12 of 49 Dr. Ravi Billa Example 4 [The exponential sequence, cont’d.] The real-valued exponential sequence x(n) = , is a basic building block in discrete-time systems. If is a positive fraction then for n 0, the sequence ( ) is a monotonically decreasing sequence. If is a negative fraction then for n 0, the sequence ( ) is an alternating sequencewith decreasing magnitudes. See MATLAB segments below. %Plot the function xn = a^n a = 0.8; n = 0: 20;xn = a.^n; subplot(2,1,1); stem(n, xn, 'b');xlabel ('n'), ylabel('x(n)'); title ('x(n) = a^n for n = 0 to 20');legend ('x(n) = (0.8)^n u(n)'); grid; % a = -0.8; n = 0: 20; xn = a.^n; subplot(2,1,2); stem(n, xn, 'b'); xlabel ('n'), ylabel('x(n)'); title ('x(n) = a^n for n = 0 to 20'); legend ('x(n) = (-0.8)^n u(n)'); grid; 0 2 4 6 8 10 12 14 16 18 20 0 0.5 1 n x(n) x(n) = an for n = 0 to 20 x(n) = (0.8)n u(n) 0 2 4 6 8 10 12 14 16 18 20 -1 -0.5 0 0.5 1 n x(n) x(n) = an for n = 0 to 20 x(n) = (-0.8)n u(n) www.jntuworld.com www.jntuworld.com
  • 13. S&S-8 (The z-transform) 13 of 49 Dr. Ravi Billa Example 5 [The exponential sequence, cont’d.] If | |> 1 then for n 0, the sequence ( ) is a monotonically increasing sequence if is positive. If, however, is negative then for n 0, the sequence ( ) is an alternating sequencewith increasing magnitudes. See MATLAB segments below. %Plot the function xn = a^n a = 1.1; n = 0: 20; xn = a.^n; subplot(2,1,1); stem(n, xn, 'b'); xlabel ('n'), ylabel('x(n)'); title ('x(n) = a^n for n = 0 to 20'); legend ('x(n) = (1.1)^n u(n)'); grid; % a = -1.1; n = 0: 20; xn = a.^n; subplot(2,1,2); stem(n, xn, 'b'); xlabel ('n'), ylabel('x(n)'); title ('x(n) = a^n for n = 0 to 20'); legend ('x(n) = (-1.1)^n u(n)'); grid; 0 2 4 6 8 10 12 14 16 18 20 0 2 4 6 8 n x(n) x(n) = an for n = 0 to 20 0 2 4 6 8 10 12 14 16 18 20 -10 -5 0 5 10 n x(n) x(n) = an for n = 0 to 20 x(n) = (1.1)n u(n) x(n) = (-1.1)n u(n) www.jntuworld.com www.jntuworld.com
  • 14. S&S-8 (The z-transform) 14 of 49 Dr. Ravi Billa Example 6 [The exponential sequence, cont’d. (Negative time)] The real-valued exponential sequence ( ) , defined for negative is another basic building block in discrete-time systems. (The minus sign in front of is chosen for a reason.) Here again there are two broad categories: | |< 1 and | |> 1. In this case, however, if is a positive fraction then, as , the sequence ( ) increases monotonically. If is a negative fraction then, as , the sequence ( ) alternates in sign while increasing in magnitude. In the MATLAB plots below note that the vertical axis ( ) is at the right edge, not the left. %Plot the function xn = -b^n b = 0.9; n = -20: -1; xn = -b.^n; subplot(2,1,1); stem(n, xn, 'b'); xlabel ('n'), ylabel('x(n)'); title ('x(n) = -b^n for n = -20 to -1'); legend ('x(n) = -(0.9)^n u(-n-1)'); grid; % b = -0.9; n = -20: -1; xn = -b.^n; subplot(2,1,2); stem(n, xn, 'b'); xlabel ('n'), ylabel('x(n)'); title ('x(n) = -b^n for n = -20 to -1'); legend ('x(n) = -(-0.9)^n u(-n-1)'); grid; -20 -18 -16 -14 -12 -10 -8 -6 -4 -2 0 -10 -5 0 n x(n) x(n) = -bn for n = -20 to -1 x(n) = -(0.9)n u(-n-1) -20 -18 -16 -14 -12 -10 -8 -6 -4 -2 0 -10 -5 0 5 10 n x(n) x(n) = -bn for n = -20 to -1 x(n) = -(-0.9)n u(-n-1) www.jntuworld.com www.jntuworld.com
  • 15. S&S-8 (The z-transform) 15 of 49 Dr. Ravi Billa If | |> 1 then, as , the sequence ( ) decreases monotonically if is positive. However, if is negative then, as , the sequence ( ) decreases in magnitude while alternating in sign. Again, in the MATLAB plots the vertical axis ( ) is at the right edge. %Plot the function xn = -b^n b = 1.1; n = -20: -1; xn = -b.^n; subplot(2,1,1); stem(n, xn, 'b'); xlabel ('n'), ylabel('x(n)'); title ('x(n) = -b^n for n = -20 to -1'); legend ('x(n) = -(1.1)^n u(-n-1)'); grid; % b = -1.1; n = -20: -1; xn = -b.^n; subplot(2,1,2); stem(n, xn, 'b'); xlabel ('n'), ylabel('x(n)'); title ('x(n) = -b^n for n = -20 to -1'); legend ('x(n) = -(-1.1)^n u(-n-1)'); grid; -20 -18 -16 -14 -12 -10 -8 -6 -4 -2 0 -1 -0.5 0 n x(n) x(n) = -bn for n = -20 to -1 x(n) = -(1.1)n u(-n-1) -20 -18 -16 -14 -12 -10 -8 -6 -4 -2 0 -1 -0.5 0 0.5 1 n x(n) x(n) = -bn for n = -20 to -1 x(n) = -(-1.1)n u(-n-1) www.jntuworld.com www.jntuworld.com
  • 16. S&S-8 (The z-transform) 16 of 49 Dr. Ravi Billa Example 7 [The sinusoidal sequence] Consider the continuous-time sinusoid x(t) ( ) and are the analog frequency in Hertz (or cycles per second) and radians per second, respectively. The sampled version is given by ( ) We may drop the from ( ) and write ( ) We may write which is the digital frequency in radians (per sample), so that ( ) Setting gives ( ) which is the digital frequency in cycles per sample. In the analog domain the horizontal axis is calibrated in seconds; “second” is one unit of the independent variable, so and are in “per second”. In the digital domain the horizontal axis is calibrated in samples; “sample” is one unit of the independent variable, so and are in “per sample”. Periodic signal The discrete-time signal x(n) is periodic if, for some integer N > 0 ( ) ( ) The smallest value of N that satisfies this relation is the (fundamental) period of the signal. If there is no such integer N, then ( )is an aperiodic signal. Example Given that the continuous-time signal ( ) is periodic, that is, ( ) ( ) for all t, and that the discrete-time signal ( ) is obtained by sampling ( ) at T-second intervals, under what conditions will ( ) be periodic? Solution If ⁄ ⁄ for integers and then ( ) has exactly N samples in L periods of ( ) and ( ) is periodic with period N. If ⁄ for integer , then ( ) ( ) for all , or ( ) ( ) (( ⁄̅̅̅̅̅̅̅) ) (( ) ) for all which implies periodicity with a period of . More generally, if ⁄ ⁄ for integers and , then ( ) ( ) for all , or ( ) ( ) (( ⁄̅̅̅̅̅̅̅̅̅) ) (( ) ) for all which again implies a period of . Thus ( ) will be periodic if is a rational number but not otherwise. Periodicity of sinusoidal sequences The sinusoidal sequence ( ) has several major differences from the continuous-time sinusoid as follows: a) The sinusoid ( ) ( ) ( ) is periodic if , that is, , is rational. If is not rational the sequence is not periodic. Replacing n with (n+N) we get www.jntuworld.com www.jntuworld.com
  • 17. S&S-8 (The z-transform) 17 of 49 Dr. Ravi Billa ( )= ( ( ))= ( ) ( )+ ( ) ( ) Clearly ( ) will be equal to ( ) – that is, the above expression reduces to ( ) – if , an integer or . The fundamental period is obtained by choosing m as the smallest integer that yields an integer value for N. For example, if = 15/25, which in reduced fraction form is 3/5, then we can choose m = 3 and get N = 5 as the (fundamental) period. In general, if is rational then = where p and q are integers. If is in reduced fraction form then the denominat or q is the period as in the above example. On the other hand if is irrational, say √ , then N will not be an integer, and thus x(n) is aperiodic. b) The sinusoidal sequences ( ) and (( ) ) for 0 ω0 2π are identical. This can be shown using the identity (( ) ) ( ) = ( ) ( ) ( ) ( ) QED Similarly, ( ) and (( ) ) are the same. Therefore in considering sinusoidal sequences for analysis purposes can be restricted to the range without any loss of generality. Example 8 [The complex exponential sequence] Obtain the discrete-time sequence corresponding to ( ) . The signal ( ) may be discretized by replacing t with nT, yielding the sequence ( ) Setting we have ( ) The sequence or is periodic if ( ) is rational. The parameters and =( ) are the digital frequency in radians/sample and cycles/sample, respectively. Note: In expressions such as ( ) or ( ) or or we shall loosely refer to ω or f as the (digital) frequency even when the signal concerned is not periodic by the definition of periodicity given above. Example9 Plot the sequences ( ) and ( ) ( ). What are their “frequencies”? Which of them is truly periodic and what is its periodicity? Solution The MATLAB program segment follows: N = 21; n = 0: N-1; % %Nonperiodic x1= 2*cos(1*n); subplot(2, 1, 1), stem(n, x1); xlabel('n'), ylabel('x1'); title('x1 = 2 cos 1n'); % www.jntuworld.com www.jntuworld.com
  • 18. S&S-8 (The z-transform) 18 of 49 Dr. Ravi Billa %Periodic x2 = 2*cos(0.2*pi*n); subplot(2, 1, 2), stem(n, x2); xlabel('n'), ylabel('x2'); title('x2 = 2 cos 0.2pi n'); 0 2 4 6 8 10 12 14 16 18 20 -2 -1 0 1 2 n x1 x1 = 2 cos 1n 0 2 4 6 8 10 12 14 16 18 20 -2 -1 0 1 2 n x2 x2 = 2 cos 0.2 n www.jntuworld.com www.jntuworld.com
  • 19. S&S-8 (The z-transform) 19 of 49 Dr. Ravi Billa Example10 Plot the sequence ( ) ( ) ( ). Solution The MATLAB program segment follows: n = [0: 30]; % %“.^” stands for element-by-element exponentiation %“.*” stands for element-by-element multiplication x3 = 2* ((0.9) .^n) .*cos(0.2*pi*n); stem(n, x3); xlabel('n'), ylabel('x3'); title('Sequence x3(n)'); The sum of two discrete-time periodic sequences is also periodic. Let x(n) be the sum of two periodic sequences, x1(n) and x2(n), with periods N1 and N2 respectively. Let p and q be two integers such that = and (p and q can always be found) Then x(n) is periodic with period N since, for all n, ( ) ( ) ( ) ( ) ( ) ( ) ( ) ( ) Definition A discrete-time signal is a sequence, that is, a function defined on the positive and negative integers. 0 5 10 15 20 25 30 -1.5 -1 -0.5 0 0.5 1 1.5 2 n x3 Sequence x3(n) www.jntuworld.com www.jntuworld.com
  • 20. S&S-8 (The z-transform) 20 of 49 Dr. Ravi Billa The sequence ( ) ( ) ( ) is a complex (valued) sequence if ( ) is not zero for all n. Otherwise, it is a real (valued) sequence. Signal descriptions such as x1(n) = 2 cos 3n or x2(n) = 3 sin (0.2πn) are an example of signal representation in functional form. Alternatively, if a signal is non-zero over a finite (small enough) interval, we can list the values of the signal as the elements of a sequence. For example x3(n) = {5, 2, – 1, 1, – 1/2, 4}  The arrow indicates the value at n = 0. We omit the arrow when the first entry represents the value for n = 0. The above sequence is a finite length sequence. It is assumed that all values of the signal not listed are zero. In the above example x(0) = 1, x(1) = –1/2, x(–4) = x(3) = 0, etc. Definition A discrete-time signal whose values are from a finite set is called a digital signal. Odd and even sequences The signal x(n) is an even sequence if ( ) ( ) for all n, and is an odd sequence if ( ) ( ) for all n. The even part of ( ) is determined as ( ) ( ) ( ) and the odd part of ( ) is given by ( ) ( ) ( ) The signal ( ) then is given by ( ) ( ) ( ). Definition A discrete-time sequence ( ) is called causal if it has zero values for n< 0, i.e., ( )= 0 for n< 0. If this condition is not satisfied the signal is non-causal. (An anticausal sequence has zero values for , i.e., ( )= 0 for .) x(n) (Even) n 0 1 2 –1 –2 –1 x(n) (Odd) n 0 2 –2 1 www.jntuworld.com www.jntuworld.com
  • 21. S&S-8 (The z-transform) 21 of 49 Dr. Ravi Billa The unit sample sequence (discrete-time impulse, aka the Kronecker delta) ( ) Whereas δ(n) is somewhat similar to the continuous-time impulse function δ(t) – the Dirac delta – we note that the magnitude of the discrete impulse is finite. Thus there are no analytical difficulties in defining δ(n). It is convenient to interpret the delta function as follows: ( ) The unit step sequence ( ) ( ) a) The discrete delta function can be expressed as the first difference of the unit step function: ( ) ( ) ( ) b) The sum from – to n of the δ function gives the unit-step: ∑ ( ) ( ) δ(n) n 0 1–1 1 1 δ(n–k) n 0 1–1 k www.jntuworld.com www.jntuworld.com
  • 22. S&S-8 (The z-transform) 22 of 49 Dr. Ravi Billa Results (a) and (b) are like the continuous-time derivative and integral respectively. c) By inspection of the graph of u(n), shown below, we can write: ( ) ( ) ( ) ( ) ∑ ( ) d) For any arbitrary sequence x(n), we have ( ) ( ) ( ) ( ) that is, the multiplication will pick out just the one value x(k). If we find the infinite sum of the above we get the sifting property: ∑ ( ) ( ) ( ) e) We can write x(n) as follows: ( ) ( ) ( ) ( ) ( ) ( ) ( ) ( ) ( ) This can be verified to be true for all n by setting in turn The above can be written compactly as n Sum up to here is zero k 0 k 0 n Sum up to here is 1 0 δ(n) u(n) n δ(n–1) 1 δ(n–2) 2 δ(n–3) 3 www.jntuworld.com www.jntuworld.com
  • 23. S&S-8 (The z-transform) 23 of 49 Dr. Ravi Billa ( ) ∑ ( ) ( ) This is a weighted-sum of delayed unit sample functions. 8.3 The z-transform For continuous-time systems the Laplace transform is an extension of the Fourier transform. The Laplace transform can be applied to a broader class of signals than the Fourier transform can, since there are many signals for which the Fourier transform does not converge but the Laplace transform does. The Laplace transform allows us, for example, to perform transform analysis of unstable systems and to develop additional insights and tools for linear time-invariant (LTI) system analysis. The z-transform is the discrete-time counterpart of the Laplace transform. The z-transform enables us to analyze certain discrete-time signals that do not have a discrete-time Fourier transform. The motivations and properties of the z-transform closely resemble those of the Laplace transform. However, as with the relationship of the continuous time versus the discrete-time Fourier transforms, there are distinctions between the Laplace transform and the z-transform. Definition The two-sided (bilateral) z-transform, X(z), of the sequence x(n) is defined as ( ) { ( )} ∑ ( ) where is the complex variable. The above power series is a Laurent series. The one–sided (unilateral) z-transform is defined as ( ) ∑ ( ) The unilateral z-transform is particularly useful in analyzing causal systems specified by linear constant-coefficient difference equations with nonzero initial conditions into which inputs are stepped. It is extensively used in digital control systems. The region of convergence (ROC) is the set of z values for which the above summation converges. In general the ROC is an annular region in the complex z-plane given by | | Im Re ROC Rx– Rx+ z-plane www.jntuworld.com www.jntuworld.com
  • 24. S&S-8 (The z-transform) 24 of 49 Dr. Ravi Billa Relationship between the z-transform and the discrete-time Fourier transform In the continuous- time situation we denote the Laplace transform of ( ) by ( ) where = is the transform variable in the complex -plane. The variable is the analog frequency in radians per second. The corresponding Fourier transform is denoted by ( ). When the Laplace transform reduces to the Fourier transform. In other words, when evaluated on the imaginary axis (i.e., for = ) in the - plane the Laplace transform reduces to the Fourier transform. There is a similar relationship between the z-transform of the sequence ( ) and its discrete- time Fourier transform. To explore this we express the complex variable z in polar form as where and are respectively the magnitude and angle of . The variable ω is the digital frequency in radians per sample. Substituting for in the definition of the -transform gives us ( )| ∑ ( )( ) ∑ [ ( )] Equivalently, ( ) ∑ [ ( )] In other words, ( ) is the Fourier transform of the sequence ( ) multiplied by the real exponential (convergence factor) . The exponential weighting may decay or grow with increasing , depending on whether is greater than or less than 1. In particular, for | | the - transform reduces to the discrete-time Fourier transform, that is, ( )| ( ) ∑ ( ) Thus, the z-transform, when evaluated on the unit circle, reduces to the discrete-time Fourier transform. In the discussion of the -transform the role of the unit circle in the -planeis similar to that of the imaginary axis in the -planein the discussion of the Laplace transform. The -transform of ( ) then is the Fourier transform of ( ) : ( ) ( ) { ( ) } ∑ [ ( )] From this equation, it is seen that, for convergence of the -transform, we require that the Fourier transform of ( ) converge. For any specific ( ), we would expect this convergence for some values of and not for others. In general, the -transform of a sequence, ( ), has a region of convergence (ROC) – the range of values of for which ( ) converges. If the ROC includes the unit circle, then the Fourier transform of ( ) also converges. jΩ σ s-plane 1 Im Re z-plane www.jntuworld.com www.jntuworld.com
  • 25. S&S-8 (The z-transform) 25 of 49 Dr. Ravi Billa 8.4 Transforms of some useful sequences Example 1 [Exponential ( )] The positive-time signal ( ) If a is a positive fraction this sequence decays exponentially to 0 as n → ∞. Substituting x(n) into the defining equation, the z-transform is { ( )} ( ) ∑ ∑( ) | | | | | | The ROC is | | | |. This X(z) is a rational function (a ratio of polynomials in z). The roots of the numerator polynomial are the zeros of X(z) and the roots of the denominator polynomial are the poles of X(z). If | | , the ROC does not include the unit circle; for such values of , the Fourier transform of ( ) does not converge. This is a right-sided sequence. A right-sided sequence has an ROC that is the exterior of a circle with radius Rx– (| | | |in this case, so the radius is | |). If the ROC is the exterior of a circle the underlying sequence is a right-sided sequence. Definition A right-sided sequence ( ) is one for which ( ) for all n < n0 where n0 is positive or negative but finite. If n0 0 then x(n) is a causal or positive-time sequence. Example 2 [Exponential ( ) (negative time)] Recall that the unit step sequence u(.) = 1 if the argument of u(.) is  0, i.e., if (–n–1)  0 or n  –1. ( ) If b> 1 this sequence decays exponentially to 0 as n → –∞. The z-transform is, Im Re a Pole at a Zero at 0 ROC, |z| > |a| (Shaded area) z-plane www.jntuworld.com www.jntuworld.com
  • 26. S&S-8 (The z-transform) 26 of 49 Dr. Ravi Billa { ( )} ( ) ∑ ( ) ∑ ∑ ( ) Let n = –m and change the limits accordingly to get, ( ) ∑ ( ) ∑( ) We added 1 in the last step above to make up for the m = 0 term within the summation. The result is, ( ) ( ) | | | | | | This is a left-sided sequence. A left-sided sequence has an ROC that is the interior of a circle, z < Rx+. In this case the ROC is z < b .If the ROC is the interior of a circle the underlying sequence is a left-sided sequence. Definition A left-sided sequence ( ) is one for which ( ) for all n  n0, where n0 is positive or negative but finite. If n0 0 then x(n) is an anticausal or negative-time sequence. Note that if b = a then the two examples above have exactly the same X(z). So what makes the difference? The region of convergence makes the difference. Example 3 [Two-sided sequence] This is the sum of the positive- and negative-time sequences of the previous two examples. ( ) ( ) ( ) Substituting into the defining equation, ( ) { ( )} ∑ [ ( ) ( )] ∑ ∑ Now, from Examples 1 and 2, ∑ | | | | ∑ | | | | So, the desired transform Y(z) has a region of convergence equal to the intersection of the two separate ROC’s | | | |and | | | |. Thus b Im Re zero pole z-plane ROC www.jntuworld.com www.jntuworld.com
  • 27. S&S-8 (The z-transform) 27 of 49 Dr. Ravi Billa ( ) (| | | |) (| | | |) ( ) ( )( ) | | | | | | The ROC is the overlap of the shaded regions, that is, the annular region between | |and| |. The two zeros are at 0 and( ) , and the two poles at a and b. If b < a the transform does not converge. In the above three examples we may express the z-transform both as a ratio of polynomials in z (i.e., positive powers) and as a ratio of polynomials in z–1 (negative powers). From the definition of the z-transform, we see that for sequences which are zero for n< 0, X(z) involves only negative powers of z. However, reference to the poles and zeros is always in terms of the roots of the numerator and Im Re a b poles zero zero(a+b)/2For |a| < |b| ROC Im Re b a poles zero zero No ROC when |b| < |a| |z| > |a| www.jntuworld.com www.jntuworld.com
  • 28. S&S-8 (The z-transform) 28 of 49 Dr. Ravi Billa denominator expressed as polynomials in z. Also, it is sometimes convenient to refer to X(z), written as a ratio of polynomials in z (i.e., positive power of z), as having poles at infinity if the degree of the numerator exceeds the degree of the denominator or zeros at infinity if the numerator is of smaller degree than the denominator. Example 4 [Finite-length sequence] Only a finite number of sequence values are non-zero, as given below. ( ) By the defining equation we have ( ) ∑ ( ) ( ) ( ) Convergence of this expression requires simply that ( ) for . Then z may take on all values except z =  if N1 is negative and z = 0 if N2 is positive. Thus the ROC is at least 0 < z < and it may include either z = 0 or z =  depending on the sign of N1 and N2. Example 5 [Unit sample ( )] { ( )} ∑ ( ) Delayed unit sample ( ) { ( )} ∑ ( ) | | | | | | Example 6 [Unit step ( )] { ( )} ∑ ( ) ∑ | | | | Example 7 [Unit step ( )(negative time)] { ( )} ∑ ( ) ∑ ( ) ∑ ∑ ( ) | | 8.5 Important properties of z-transforms The proofs are easily obtained by using the definition of the z-transform and transformations in the summation. www.jntuworld.com www.jntuworld.com
  • 29. S&S-8 (The z-transform) 29 of 49 Dr. Ravi Billa (1) Linearity If { ( )} ( ) with { | | } and { ( )} ( ) with { | | }, then { ( ) ( )} ( ) ( ) with ROC at least the overlap of the ROC’s of ( ) and ( ). If there is any pole-zero cancellation due to the linear combination, then the ROC may be larger. (2) Translation (Time-shifting) If { ( )} ( ) with { | | } then { ( )} ( ) with the same ROC except for the possible addition or deletion of z = 0 or z = ∞ due to . Example 1 Given x(n) = {1, 2} and x2(n) =x(n+2) find X(z) and X2(z) and their respective ROCs. ( ) , ROC: entire z-plane except z = 0; ( ) ( ) , ROC: entire z-plane except z = ∞. (3) Multiplication by a complex exponential sequence (Scaling in the z-domain) If { ( )} ( ) with { | | } then { ( )} ( )| ( ) with {| | | | | | }. Example 2 Given ( )= {1, 2} and ( ) ( ) ( ) find X(z) and X2(z) and their respective ROCs. (4) Multiplication by a ramp (Differentiation in the z-domain) If { ( )} ( ) with , then { ( )} ( ) Example 3 Given ( )= {1, 2} and ( ) ( ) ( ) find ( )and ( ) and their respective ROCs. { ( )} {( ) ( )} { ( )} { ( )} { [ ( )]} (5) Time reversal If { ( )} ( )with { | | } then { ( )} ( ) with {( ) | | ( )}. Example 4 Given ( ) ( ) and ( ) ( ) determine ( ). ( ) ( ) ( ), ( ) , ROC: 0.5 <|z| ʓ-1{ )( 1 zX } = x(–n); x2(n) = ʓ-1{ )( 1 zXz }= x(–(n+1)).= ))1((2 ))1((  nun (6) Convolution in time domain leads to multiplication in frequency domain Given { ( )} ( ) with { }and { ( )} ( ) with { } and ( ) ( ) ∑ ( ) ( ) then { ( ) ( )} ( ) ( ) { } (7) Multiplication in time domain leads to convolution in frequency domain If { ( )} ( ) with { | | } and { ( )} ( ) with { | | } then { ( ) ( )} ∮ ( ) ( ) www.jntuworld.com www.jntuworld.com
  • 30. S&S-8 (The z-transform) 30 of 49 Dr. Ravi Billa with { | | } where is a complex contour integral and is a closed contour in the intersection of the ROCs of ( ) and ( ). (8) Initial Value Theorem If x(n) is a causal sequence with z transform X(z), then x(0)= )(lim zX z  (9) Final Value Theorem If { ( )} ( ) and the poles of ( ) ( ) are all inside the unit circle then the value of x(n) as n→ is given by ( )| [( ) ( )] Some also give this as ( )| [( ) ( )] Example Can the final value theorem be applied to the following z-transform? ( ) ( )( ) Solution For the final value theorem to be applicable ( ) ( ) must not have any poles on oroutside the unit circle. Here ( ) ( ) ( ) ( )( ) Since there is a pole at , the final value theorem does not apply. Because of the pole at , ( ) is unbounded as . 8.6 Transforms of some useful sequences, cont’d. Example 1 [Unit ramp ( )] If { ( )} ( ) then { ( )} ( ) ( ) {( ) } ( ) ( ) | | ( ) Example 2 [Sinusoid ( ) ( )] { ( )} ∑ ( ) ∑ ( ) (∑( ) ∑( ) ) ( ) | | | | ( ) ( ( ) ( ) ( ) ) ( ( ) ( ) ) www.jntuworld.com www.jntuworld.com
  • 31. S&S-8 (The z-transform) 31 of 49 Dr. Ravi Billa Using the identities we have { ( )} | | Example 3 [ ( ) ( )] As an extension of the above example and using property #3 { ( )} | ( ) ( ) ( ) ( ) | | Alternative: We may use the defining equation { ( )} ∑ ( ) ∑ ( ) ( ) Example 4 [Cosinusoid ( ) ( )] Using the relation ( )⁄ and a procedure similar to that for the sinusoid we get { ( )} ∑ ( ) ∑ ( ) … { ( )} | | Example 5 [ ( ) ( )] As an extension of the above example and using property #3, { ( )} | ( ) ( ⁄ ) ( ⁄ ) ( ⁄ ) ( ⁄ ) | | Alternative: We may use the defining equation { ( )} ∑ ( ) ∑ ( ) ( ) www.jntuworld.com www.jntuworld.com
  • 32. S&S-8 (The z-transform) 32 of 49 Dr. Ravi Billa Common z-transform pairs # Sequence z-transform ROC 1 ( ) 1 Entire z-plane 2 ( ) | | 3 ( ) | | 4 ( ) | | 5 ( ) | | 6 ( ) ( ) | | 7 ( ) ( ) | | 8 ( ) ( ) | | 9 ( ) ( ) | | 10 ( ) ( ) | | 8.7 Region of convergence (ROC) Given a discrete-time sequence ( ) which may be a signal or an impulse response we denote its z- transform as ( ). The transform will be a rational function whenever ( ) is a linear combination of real or complex exponentials. In other words, it may be expressed as either a ratio of polynomials in or as a ratio of polynomials in . It is characterized by a unique region of convergence. When the z-transform is expressed as a ratio of polynomials in we define its poles and zerosin terms of the roots of the denominator and numerator, respectively. Also, it is sometimes convenient to refer to the transform as having poles at infinity if the degree of the numerator exceeds that of the denominator or zeros at infinity if the numerator is of lesser degree than the denominator. Example 1 [Pole-zero plot and ROC] Give the pole-zero plot for ( ) Assuming that ( ) is causal indicate the ROC of ( ). Solution The denominator has roots (poles) at √ ( ) √( ) ( )( ) ( ) √ √ There is a zero at z = 0. Further, since the denominator degree is greater than the numerator degree by 1 it is clear that ( )= 0, so that there is an additional zero at z = ∞. www.jntuworld.com www.jntuworld.com
  • 33. S&S-8 (The z-transform) 33 of 49 Dr. Ravi Billa In MATLAB the transfer function is specified as a ratio of polynomials in 1 z ( ) The numerator coefficients, { }and the denominator coefficients { }are specified as the two vectors b = [0, 1] and a = [1, -1, -1]. %Pole-zero plot b = [0, 1]; a = [1, -1, -1]; zplane (b, a) In the pole-zero plot, a pole is marked by an x and a zero by a little circle. With ( ) ( )( ) we have ( ) and the ROC = {| | } {| | } = {| | }. Example 2 [Pole-zero plot and ROC] Give the pole-zero plot for ( ) Assuming that ( ) is causal indicate the ROC of ( ). Solution From -1 -0.5 0 0.5 1 1.5 -1 -0.8 -0.6 -0.4 -0.2 0 0.2 0.4 0.6 0.8 1 Real Part ImaginaryPart www.jntuworld.com www.jntuworld.com
  • 34. S&S-8 (The z-transform) 34 of 49 Dr. Ravi Billa ( ) we can see that there are 9 poles at z = 0 and 8 zeros at sundry places and an additional zero at z = ∞ owing to the denominator degree being greater than the numerator degree by 1. For the MATLAB segment the numerator and denominator coefficients are taken from ( ) %Pole-zero plot b = [0: 9]; a = [1, 0]; zplane (b, a) In the pole-zero plot, the pole at is marked by an x with a 9 alongside it indicating its multiplicity. The zeros are indicated by little circles. Since ( ) contains only negative powers of the ROC excludes = 0 and is given by ROC = {| | } Properties of the ROC The impulse response and its z-transform are usually denoted by ( ) and ( ), respectively. However, in what follows we use the symbols ( ) and ( ) to represent any signal including input, output and impulse response. (1) If ( ) is a right-sided sequence, the ROC of ( ) is the exterior of a circle centered at the origin with radius equal to the largest (in magnitude) of the poles of ( ). All other poles are inside this circle. -1.5 -1 -0.5 0 0.5 1 1.5 -1 -0.5 0 0.5 1 9 Real Part ImaginaryPart www.jntuworld.com www.jntuworld.com
  • 35. S&S-8 (The z-transform) 35 of 49 Dr. Ravi Billa (2) If ( ) is a left-sided sequence, the ROC of ( ) is the interior of a circle centered at the origin with radius equal to the smallest (in magnitude) of the poles of ( ).All other poles are outside this circle. (3) If ( ) is a two-sided sequence, the ROC of ( ), if it exists, is an annular ring centered at the origin.The inner boundary of the ROC is a circle of radius equal to the largest (in magnitude) of the poles of ( ) arising from the right-sided sequences that make up ( ); and the outer boundary is a circle of radius equal to the smallest (in magnitude) of the poles of ( ) arising from the left-sided sequences that make up ( ). (4) If ( ) is a finite-length sequence, the ROC of ( ) is the entire -plane, except possibly = 0 and/or = ∞. = 0 is excluded if there is a non-zero ( ) value for ; = ∞ is excluded if there is a non-zero ( ) value for . (5) The ROC does not contain any poles. 8.8 Inverse z-transform by partial fractions (Aside) Comparison of inverse z-transform methods A limitation of the power series method is that it does not lead to a closed form solution (although this can be deduced in simple cases), but it is simple and lends itself to computer implementation. However, because of its recursive nature care should be taken to minimize possible build-up of numerical errors when the number of data points in the inverse z-transform is large, for example by using double precision. Both the partial fraction method and the inversion integral method require the evaluation of residues albeit performed in different ways. The partial fraction method requires the evaluation of the residues of ( )or { ( ) }. The complex inversion integral requires the evaluation of the residues of ( ) . In many instances evaluation of the complex inversion integral is needlessly difficult and involved. Both the partial fraction method and the inversion integral method lead to closed form solutions. The main disadvantage is having to factorize the denominator polynomial of X(z) when it is of order greater than 2. Another disadvantage is multiple order poles and the resulting differentiation(s) when determining residues. The partial fraction method directly generates the coefficients of parallel structures for digital filters. The inversion integral method is widely used in the analysis of quantization errors in discrete- time systems. (End of Aside) As in Laplace transforms, in order to expand a rational function into partial fractions, the degree of the numerator should be less than the degree of the denominator – proper fraction. If it is not then we perform long division as below. As a result ( )is the quotient and ( )is the remainder with degree less than that of ( ). ( ) ( ) ( ) ( ) ( ) ( ) Long Division Q(z) ←Quotient Denominator→ D(z) N(z) ←Numerator --- N1(z) ←Remainder www.jntuworld.com www.jntuworld.com
  • 36. S&S-8 (The z-transform) 36 of 49 Dr. Ravi Billa The long division is done until we get a remainder polynomial ( )whose degree is less than the degree of the denominator ( ). We then obtain x(n) as ( ) { ( )} { ( )} { ( ) ( ) } Since ( ) ( )⁄ is a proper fraction it can be expanded into partial fractions. The overall inverse transform is obtained by looking up a table of z-transform pairs. However, there is an alternative available in the case of z-transforms which is not available in Laplace transforms. This is a result of the fact that z-transforms are characterized by a z in the numerator (as can be verified by looking at a table of z-transforms). Therefore, instead of expanding ( )we may, instead, expand [ ( )⁄ ] into partial fractions giving ( ) so that ( )is given by ( ) This can be inverted by a simple look-up of a table of transforms. Note also that in some cases ( ) ( ) ( ) may not be a proper fraction but [ ( )⁄ ] is and, therefore, this method obviates the need for long division of ( ) by ( ). (In still other cases even [ ( )⁄ ] may not be a proper fraction. See later under “General procedure for partial fraction expansion”.) Example 1 (See also long division later). Find the inverse z-transform, using partial fractions, of ( ) ( ) ( ) State your assumptions. Solution This is not a proper fraction since the degree of the numerator is not less than the degree of the denominator. However, [ ( )⁄ ] is a proper fraction. ( ) ( )( ) whose partial fraction expansion is obtained as below: ( ) ( )( ) ( ) | ( ) ( ) ( ) | ( ) ( ) ( ) ( ) By looking up a table of z-transforms the inverse z-transform is { ( )} ( ) ( ) ( ) Note that we are giving here the causal solution that corresponds to a ROC |z| > 2 (not 1 < |z| < 2 or |z| < 1) so that x(n) is a right-sided sequence. www.jntuworld.com www.jntuworld.com
  • 37. S&S-8 (The z-transform) 37 of 49 Dr. Ravi Billa The alternative method is to divide ( ) by ( )as below (as is standard practice in Laplace transforms). Note that in this long division the numerator and denominator polynomials are arranged in the order of decreasing powers of z. There are three other ways (all of them wrong) of arranging the two polynomials for the long division. Long Division 2 ←Quotient Denominator→ z2 –3z + 2 2z2 – 3z ←Numerator 2z2 – 6z + 4 3z – 4 ←Remainder Thus ( ) can be expressed as ( ) ( )( ) ( ) where ( ) ( )( ) is a proper fraction and can be expanded into partial fractions as below: ( ) ( )( ) ( ) ( ) Solving for A and B we get A = 1 and B = 2, so that X(z) may be written ( ) ( ) ( ) Taking the inverse z-transform we get ( ) { ( )} { ( ) ( ) } { } { } { } A term like { } is handled by writing ( )as ( ). We know that { } ( ), so { } { } ( ) Similarly { } { } ( ) Thus ( ) ( ) ( ) ( ) This can be verified to be equivalent to ( ) ( ) ( ) obtained earlier. Example 2 Find the inverse transform of ( ) where the ROC is (a) |z|> 1, (b) |z|< (1/3), (c) (1/3) <|z|< 1. Solution The three possible regions of convergence are shown below. The example illustrates that the inverse transform, x(n), is unique only when the ROC is specified. www.jntuworld.com www.jntuworld.com
  • 38. S&S-8 (The z-transform) 38 of 49 Dr. Ravi Billa (a) For ROC ≡ |z|> 1 ( ) ( ) ( ) ( ( ) ( )) ( )( ( ⁄ )) ( ⁄ ) ( ( ⁄ )) | and ( ) | ( ) ( ⁄ ) ( ⁄ ) ( ⁄ ) ( ) ( ⁄ ) ( ⁄ ) ( ⁄ ) The inverse is ( ) { ( )} { ( ⁄ ) ( ⁄ ) ( ⁄ ) } { ( ⁄ ) } { ( ⁄ ) ( ⁄ ) } The ROC is outside the largest pole signifying a right-sided sequence for each pole. The inverse becomes ( ) ( ⁄ )( ) ( ) ( ⁄ )( ⁄ ) ( ) {( ⁄ ) ( ⁄ )( ⁄ ) } ( ) (b) For ROC ≡ |z|< (1/3). The partial fraction expansion does not change. Since the ROC is inward of the smallest pole, x(n) consists of two negative-time sequences. ( ) { ( )} { ( ⁄ ) ( ⁄ ) ( ⁄ ) } { ( ⁄ ) } { ( ⁄ ) ( ⁄ ) } ( ) ( ) ( ( ) ) ( ) { ( ) } ( ) ( ⁄ ){ ( ⁄ ) } ( ) (a) (b) (c) www.jntuworld.com www.jntuworld.com
  • 39. S&S-8 (The z-transform) 39 of 49 Dr. Ravi Billa (c) For ROC ≡ (1/3) <|z|< 1. The partial fraction expansion stays the same. The pole at z = 1 corresponds to a negative-time sequence (left-sided sequence) while the pole at z = 1/3 gives a positive- time sequence (right-sided sequence). This is the only possibility. ( ) { ( )} { ( ⁄ ) ( ⁄ ) ( ⁄ ) } { ( ⁄ ) } { ( ⁄ ) ( ⁄ ) } ( ) ( ) ( ) ( ) ( ) ( ) ( ) The overall result is a two-sided sequence. Example 3 (Sometimes there is no z in the numerator to factor out, but we still divide X(z) by z as in this example.) Find ( )for ( ) | | Solution ( ) ( ) ( )( ( ⁄ )) ( ⁄ ) ( )( ( ⁄ )) | ( )( ⁄ ) ( ( ⁄ )) | ( ) | ( ) ( ⁄ ) ( ) ( ⁄ ) ( ) { ( )} { ( ⁄ ) } Re Im 1 (c) ⁄ | | 1/3 www.jntuworld.com www.jntuworld.com
  • 40. S&S-8 (The z-transform) 40 of 49 Dr. Ravi Billa ( ) ( ) ( ) ( ) Example 4 Find the inverse z-transform of ( ) ( ) Assume that ( ) is causal. Solution The roots of the quadratic in the denominator are given by √ ( ) √( ) ( )( ) ( ) √ √ ( ) ( ) ( )( ) ( ) ( ) ( )( ) ( ) ( )( ) | ( ) ( ) | ( ) ( ) | ( ) ( ⁄ ) ( ) ( ⁄ ) ( ) { ( )} { ( ⁄ ) } ( ) ( ) ( ) ( ) ( ) Example 5 Find the inverse z-transform of ( ) ( ) ( ) | | Solution ( ) ( ) ( ) ( ⁄ ) ( ⁄ ) ( ) ( ) | ( ) | www.jntuworld.com www.jntuworld.com
  • 41. S&S-8 (The z-transform) 41 of 49 Dr. Ravi Billa ( ) | ( ) ( ⁄ ) ( ⁄ ) ( ) ( ⁄ ) ( ⁄ ) ( ) ( ) ( ) ( ) ( ) ( ) Example 6 Find the inverse of ( ) ( ) ( ) ⁄ | | Solution ( ) ( ) ( ) ( ) ( ) There is a pole at z = ∞. The numerator degree is 3 and is greater than the denominator degree. By long division we reduce the numerator degree by 2 so that the resulting numerator degree is less than that of the denominator degree. ( ) ( ) ( ) ( ( ⁄ ) ( ⁄ ) ( ⁄ ) ( ⁄ ) ) (Note that in the long division leading to the above result the numerator and denominator polynomials are arranged in the order of decreasing powers of z. There are three other ways (all of them wrong) of arranging the two polynomials for the long division.) The proper fraction part can now be expanded into partial fractions: ( ⁄ ) ( ⁄ ) ( ⁄ ) ( ⁄ ) ( ⁄ ) ( ⁄ ) ( ⁄ ) ( ⁄ ) ( ⁄ ) | ( ⁄ ) ( ⁄ ) ( ⁄ ) | ( ) ( ) ( ⁄ ) ( ) ( ⁄ ) ( ) ( ) ( ⁄ ) ( ) ( ⁄ ) ( ) ( ) ( ) { ( ) ( ) } ( ) The answer has values for n = –1 and n = –2 due to the double pole at z = ∞. The resulting x(n) is not a causal sequence. Example 7 Assuming that ( ) is a causal system function, prove the following independently of each other. www.jntuworld.com www.jntuworld.com
  • 42. S&S-8 (The z-transform) 42 of 49 Dr. Ravi Billa (a) ( ) ( ⁄ ) ( ) ( ⁄ ) ( ) (b) ( ) ( ) ( ) ( ) (c) ( ) ( ) ( ) ( ) Solution (a) Expand [ ( )⁄ ] into partial fractions ( ) ( ⁄ ) ( ⁄ ) (b) Write ( ) ( ) ( )⁄ as ( ) (c) By long division ( ) Inverse z-transform when there are repeated roots With repeated roots, that is, a -th order pole at we have ( )in the form ( ) ( ) | | The table below gives the inverse z-transforms for several values of k and for the general case of arbitrary k. Repeated Roots ( ) ( ) { ( )} | | | | ( ) ( ) ( ) ( ) ( ) ( ) ( ) ( ) ( )( ) ( ) … … ( ) ( )( ) ( ̅̅̅̅̅̅̅) ( ) ̅̅̅̅̅̅ ( ) Example 10 Find the inverse of ( ) ( ) ( ) | | Solution ( ) ( ) ( ) ( ) ( ) ( ) ( ) ( ) | ( ) ( ) www.jntuworld.com www.jntuworld.com
  • 43. S&S-8 (The z-transform) 43 of 49 Dr. Ravi Billa ( ) | ( ) ( ) ( ) ( ) | ( ) ( ) ( ) { ( ( ) )}| { ( ( ) )}| { ( ) ( ) ( ) ( ) ( ) }| ( ) ( ) ( ) ( ) ( ) ( ) ( ) ( ) ( ) ( ) ( ) ( ) ( ) Taking the inverse z-transform, ( ) { ( )} { ( ) ( ) ( ) } { } { ( ) } { ( ) } { ( ) } ( ) ( ) ( ) ( ) ( ) ( ) ( ) Other possibilities If we choose to expand ( ), rather than [ ( ) ], into partial fractions, we need to perform long division to reduce the degree of the numerator by 1 resulting in ( ) ( ) where ( )is the proper fraction part of the above ( ) ( ) { ( )} ( ) { ( )} Either ( ) itself or [ ( ) ] may be expanded into partial fractions. Example 11 Find the inverse of www.jntuworld.com www.jntuworld.com
  • 44. S&S-8 (The z-transform) 44 of 49 Dr. Ravi Billa ( ) ( ) ( ) | | Solution ( ) ( ) ( ) ( ) ( ) ( ) ( ) ( ) | ( ) ( ) ( ) { ( ( ) )}| ( ) { ( ( ) )}| ( ) | ( ) ( ) ( ) ( ) ( ) ( ) ( ) ( ) ( ) ( ) ( ) ( ) ( ) { ( )} { ( ) ( ) ( ) ( )} { ( ) } { ( ) } { ( ) } { ( ) } ( ) ( ) ( ) ( ) ( ) ( ) ( ) ( ) ( ) The u(n) may be factored out etc. Example 12 [Differentiation] Find the inverse of ( ) ( ) | | | | Solution This does not involve partial fractions. Rather we use the property dealing with multiplication by a ramp in the time domain (differentiation in the z-domain). According to this property if { ( )} ( ) with , then { ( )} ( ) Note that ( ) ( ) ( ) Here www.jntuworld.com www.jntuworld.com
  • 45. S&S-8 (The z-transform) 45 of 49 Dr. Ravi Billa ( ) { ( )} ( ) ( ) ( ) [ ( ) ( ) ] ( ) ( ) ( ) { { ( ) }} The ROC | | | | implies a right-sided sequence, so that { ( ) } { ( ) } [ ( )]| ( ) ( ) Thus we have ( ) { ( )} { ( )} Therefore ( ) ( ) ( ) ( ) ( ) www.jntuworld.com www.jntuworld.com
  • 46. S&S-8 (The z-transform) 46 of 49 Dr. Ravi Billa 8.9 Inverse z-transform by power series expansion (long division) If the z-transform is expressed as a rational function (a ratio of polynomials in or ) we can use long division (aka synthetic division) to expand it into a power series. If the transform is expressed as an irrational function we can use the appropriate power series expansion formula (if) available in mathematical tables such as the CRC Tables. Note that if the transform is expressed as an irrational function then the partial fraction expansion method of inversion won’t work. By definition the z-transform of the sequence x(n) is given by ( ) ∑ ( ) ( ) ( ) ( ) ( ) This is a power series (Laurent series). So by long division we obtain the power series expansion of X(z) and then, by comparison with the power series definition given above, we can identify the sequence x(n). In particular the coefficient of z–k is the sequence value x(k). The method is useful in obtaining a quick look at the first few values of the sequence x(n). This approach does not assure an analytical solution. The ROC will determine whether the series has positive or negative exponents. For right-sided sequences the X(z) will be obtained with primarily negative exponents, while left-sided sequences will have primarily positive exponents. For an annular ROC, a Laurent expansion would give both positive- and negative-time terms. This last possibility is illustrated in the example below by taking a little help from partial fractions. Example 1 Find the inverse transform, by long division, of ( ) ( )( ) where the ROC is (a)| | , (b)| | , (c) | | . Solution (a) ROC is| | . We expect a right-sided sequence, with predominantly negative exponents of . For the long division arrange numerator and denominator as decreasing powers of and then divide; or as increasingly negative power of i.e., and then divide. z-plane Im Re 2 ROC, |z| > |2| (Shaded area) 1 www.jntuworld.com www.jntuworld.com
  • 47. S&S-8 (The z-transform) 47 of 49 Dr. Ravi Billa 2 +3z–1 +5z–2 + 9z–3 + … ←Q(z) D(z)→ z2 –3z + 2 2z2 – 3z ←N(z) 2z2 – 6z + 4 3z – 4 3z – 9 + 6z–1 5 – 6z–1 5 – 15z–1 + 10z–2 9z–1 – 10z–2 9z–1 – 27z–2 + 18z–3 17z–2 – 18z–3 … Thus ( ) . By comparison with the defining equation ( ) ( ) ( ) ( ) ( ) we see that the sequence values are ( ) ( ) , or ( ) for n< 0, and ( ) , ( ) , ( ) , etc. Alternatively, it is also possible to write X(z) as a ratio of polynomials in z–1 ( ) Note that the polynomials are written in the order of increasing negative powers of z, that is, z–1 . Long division gives (the same answer as obtained earlier): 2 +3z–1 + 5z–2 + 9z–3 + … ←Q(z) D(z)→ 1– 3z–1 + 2z–2 2 – 3z–1 ←N(z) 2 – 6z–1 + 4z–2 3z–1 – 4z–2 3z–1 – 9z–2 + 6z–3 5z–2 – 6z–3 … Solution (b) The ROC is |z|< 1. We expect a left-sided sequence with predominantly positive exponents of z. For the long division the polynomials are written in the order of increasing powers of z (or decreasingly negative powers of z, i.e., z–1 ). –(3/2)z – (5/4)z2 – (9/8)z3 – … ←Q(z) Im Re 21 ROC www.jntuworld.com www.jntuworld.com
  • 48. S&S-8 (The z-transform) 48 of 49 Dr. Ravi Billa D(z)→ 2 – 3z + z2 –3z + 2z2 ←N(z) –3z+(9/2)z2 – (3/2)z3 – (5/2)z2 + (3/2)z3 – (5/2)z2 + (15/4)z3 –(5/4)z4 – (9/4)z3 + (5/4)z4 – (9/4)z3 + (27/8)z4 – (9/8)z5 Thus X(z) = –(3/2)z– (5/4)z2 – (9/8)z3 – … = …– (9/8)z3 – (5/4)z2 – (3/2)z. By comparing with the defining equation X(z) = …x(–3)z3 + x(–2)z2 + x(–1)z + x(0) + x(1)z–1 +… we see that the sequence is given by x(–1) = –3/2, x(–2) = –5/4, x(–3) = –9/8, …etc., and x(n) = 0 for n 0 The other way of long division is shown below: –(3/2)z – (5/4)z2 – (9/8)z3 – … ←Q(z) D(z)→ 2z–2 – 3z–1 + 1 – 3z–1 + 2 ←N(z) – 3z–1 + (9/2) – (3/2)z … (Omit) Solution (c) The ROC is 1 <|z|< 2. We expect a two-sided sequence with both positive and negative exponents of z. Looking at the pole-zero configuration, the pole at z =1 implies a right-sided sequence while the pole at z = 2 a left-sided sequence. Obviously just a single long division cannot give both the left-sided and the right-sided sequences simultaneously. We shall obtain the partial fraction expansion first and then proceed with the long divisions to obtain the sequences separately. These two sequences are then combined (added) into one sequence to get the solution. Note that once we use partial fractions the utility of long division is nullified. We do this only to illustrate the method of long division. ( ) ( )( ) Re Im ROC 21 www.jntuworld.com www.jntuworld.com
  • 49. S&S-8 (The z-transform) 49 of 49 Dr. Ravi Billa ( ) | ( ) ( ) ( ) | ( ) ( ) ( ) ( ) ( ) ( ) ( ) ( ) For the term ( )⁄ we have a right-sided sequence given by long division thus: 1 + z –1 + z –2 + z –3 +…… ←Q(z) D(z)→ z – 1 z ←N(z) z – 1 1 1 – z–1 z–1 z–1 – z–2 z–2 … The corresponding sequence is ( ) For the term ( )⁄ we have a left sided sequence –(1/2)z – (1/4)z2 – (1/8)z3 ……. ←Q(z) D(z)→ –2 + z z ←N(z) z – (1/2)z2 (1/2)z2 (1/2)z2 – (1/4)z3 (1/4)z3 (1/4)z3 – (1/8)z4 (1/8)z4 … The corresponding sequence is ( ) The complete sequence is then ( ) ( ) ( ) (End of Omit) www.jntuworld.com www.jntuworld.com